20.01.2015 Aufrufe

mathe-lmu.de - Mathematisches Institut - Ludwig-Maximilians ...

mathe-lmu.de - Mathematisches Institut - Ludwig-Maximilians ...

mathe-lmu.de - Mathematisches Institut - Ludwig-Maximilians ...

MEHR ANZEIGEN
WENIGER ANZEIGEN

Erfolgreiche ePaper selbst erstellen

Machen Sie aus Ihren PDF Publikationen ein blätterbares Flipbook mit unserer einzigartigen Google optimierten e-Paper Software.

Nr. 21 – Januar 2010<br />

Mathe-LMU.<strong>de</strong><br />

För<strong>de</strong>rverein Mathematik in Wirtschaft, Universität und<br />

Schule an <strong>de</strong>r <strong>Ludwig</strong>-<strong>Maximilians</strong>-Universität München e.V.<br />

Hochschulkarrieren im Ausland - Seiten 14 bis 18<br />

Der Zufall –<br />

ein Helfer und kein Störenfried - Seite 25


2<br />

Liebe Leserinnen und Leser,<br />

Liebes Vereinsmitglied,<br />

Mathematiker können in vielen Branchen <strong>de</strong>r<br />

Industrie tätig sein. In diesem Heft fin<strong>de</strong>n Sie<br />

drei Berichte von unseren Stu<strong>de</strong>nten über ihre<br />

Arbeitserfahrungen bei Münchener Rück, Siemens<br />

AG und Société Générale Securities<br />

Services.<br />

Man kann nach <strong>de</strong>m Abschluss <strong>de</strong>s Studiums<br />

auch im universitären Bereich bleiben<br />

und eine wissenschaftliche Karriere einschlagen.<br />

In dieser Ausgabe berichten wir speziell<br />

über Wer<strong>de</strong>gänge von ehemaligen Stu<strong>de</strong>nten<br />

unseres <strong>Institut</strong>es an ausländischen Universitäten,<br />

und zwar von Dr. Ulrich Berger (Swansea<br />

University) und Prof. Dr. Markus Schmidmeier<br />

(Florida Atlantic University).<br />

Beachten Sie bitte <strong>de</strong>n Hinweis auf unsere<br />

diesjährige „Mathematik am Samstag“ auf<br />

Seite 19.<br />

Vitali Wachtel<br />

Titelbild: Auf Seite 23 geben wir eine<br />

Lösungsskizze zur Aufgabe „Würfel färben“<br />

von Heft 20. Das Titelbild zeigt eine <strong>de</strong>r 39<br />

Lösungen, aufgenommen vor einem Spiegel.<br />

Übrigens ist dies die einzige Lösung, die invariant<br />

ist unter Spiegelungen an einer Ebene.<br />

Impressum <strong>mathe</strong>-<strong>lmu</strong>.<strong>de</strong><br />

Herausgeber För<strong>de</strong>rverein Mathematik<br />

in Wirtschaft, Universität und Schule an <strong>de</strong>r<br />

<strong>Ludwig</strong>-<strong>Maximilians</strong>-Universität München e.V.,<br />

<strong>Mathematisches</strong> <strong>Institut</strong>, Universität München,<br />

Theresienstr. 39, 80333 München<br />

fmwus@<strong>mathe</strong>matik.uni-muenchen.<strong>de</strong><br />

Konto: 1267532, Bankleitzahl 700 500 00,<br />

Bayerische Lan<strong>de</strong>sbank<br />

ViSdP Vitali Wachtel, <strong>Mathematisches</strong> <strong>Institut</strong>,<br />

Universität München, Theresienstr. 39<br />

80333 München, Tel. 2180-4488<br />

wachtel@<strong>mathe</strong>matik.uni-muenchen.<strong>de</strong><br />

auf Anregung <strong>de</strong>s erweiterten Fakultätsrats<br />

erfolgt die Verabschiedung <strong>de</strong>r Studieren<strong>de</strong>n,<br />

die an <strong>de</strong>r LMU München einen Abschluss<br />

in Mathematik o<strong>de</strong>r Wirtschafts<strong>mathe</strong>matik<br />

erworben haben, seit heuer nicht mehr als<br />

ein Programmpunkt beim „Tag <strong>de</strong>r Fakultät“,<br />

son<strong>de</strong>rn im Rahmen einer eigenen Veranstaltung.<br />

Die Premiere für die Absolventinnen<br />

und Absolventen <strong>de</strong>s aka<strong>de</strong>mischen Jahres<br />

2008/09 fand am Nachmittag <strong>de</strong>s 21. November<br />

2009 in Zusammenarbeit <strong>de</strong>s Mathematischen<br />

<strong>Institut</strong>s mit <strong>de</strong>m För<strong>de</strong>rverein Mathematik<br />

statt. Nach humorvollen Ansprachen<br />

von Professor Franz Merkl als Vorsitzen<strong>de</strong>m <strong>de</strong>s<br />

Prüfungsausschusses und Vorstandsmitglied<br />

im För<strong>de</strong>rverein sowie Herrn Jan Wi<strong>de</strong>nmann<br />

als Vertreter <strong>de</strong>r frischgebackenen Alumni und<br />

umrahmt von stimmungsvoller Musik durch<br />

das Dorothea Klarinetten-Trio wur<strong>de</strong>n die insgesamt<br />

43 anwesen<strong>de</strong>n Absolventinnen und<br />

Absolventen geehrt. Die rund einstündige Veranstaltung<br />

klang mit einem kleinen Sektempfang<br />

aus, wo dann noch ausgiebig die Gelegenheit<br />

ergriffen wur<strong>de</strong>, sich zum einen über<br />

das gemeinsame erfolgreich abgeschlossene<br />

Studium und zum an<strong>de</strong>ren auch über die<br />

ersten Schritte in die berufliche Zukunft auszutauschen.<br />

Die durchwegs positive Resonanz<br />

auf diese Premiere ist sicher eine Ermunterung<br />

zur Fortsetzung in diesem Jahr.<br />

Ihr Erwin Schörner<br />

Redaktion Katharina Belaga, Bernhard Emmer,<br />

Daniel Rost, Ingrid Schehrer,<br />

Erwin Schörner, Katharina Schüller,<br />

Heinrich Steinlein, Vitali Wachtel<br />

Auflage 5000<br />

Layout Gerhard Koehler, München,<br />

kws@kws-koehler.<strong>de</strong><br />

Druck Siller Offsetdruck, Künzelsau<br />

Die Redaktion bedankt sich bei <strong>de</strong>n Firmen, die<br />

mit ihren Anzeigen die Herausgabe dieser Zeitung<br />

ermöglichten. Wir bitten die Leser um<br />

freundliche Beachtung <strong>de</strong>r Anzeigen.


Berichte aus <strong>de</strong>m Mathematischen <strong>Institut</strong><br />

3<br />

Studienangebot und Einschreibung Das<br />

Studienangebot <strong>de</strong>s Mathematischen <strong>Institut</strong>s<br />

erfreut sich auch im Wintersemester<br />

2009/10 wie<strong>de</strong>r sehr großer Beliebtheit bei<br />

<strong>de</strong>n Studieren<strong>de</strong>n; ein stichhaltiger Beleg<br />

hierfür ist die auch für dieses Studienjahr<br />

sehr erfreuliche Zahl an Neueinschreibungen<br />

in einen <strong>mathe</strong>matischen Studiengang <strong>de</strong>r<br />

LMU München. Das Diagramm zeigt <strong>de</strong>utlich,<br />

dass sich die Zahl <strong>de</strong>r Immatrikulationen<br />

mit insgesamt 465 Studienanfängerinnen<br />

und Studienanfängern auf <strong>de</strong>m hohen<br />

Niveau <strong>de</strong>s Vorjahres stabilisiert hat. Umso<br />

bedauerlicher ist <strong>de</strong>r Umstand, dass <strong>de</strong>r vom<br />

Mathematischen <strong>Institut</strong> zum Wintersemester<br />

2009/10 geplante offizielle Start eines eigenen<br />

Bachelorstudiengangs Wirtschafts<strong>mathe</strong>matik<br />

wegen <strong>de</strong>r immer noch ausstehen<strong>de</strong>n<br />

Genehmigung durch die Rechtsabteilung um<br />

ein Jahr verschoben wer<strong>de</strong>n musste.<br />

Das Mathematische <strong>Institut</strong> hat bei <strong>de</strong>r durch<br />

die Neufassung <strong>de</strong>r Lehramtsprüfungsordnung<br />

I vorgeschriebenen Modularisierung<br />

auch <strong>de</strong>r Lehramtsstudiengänge eine Vorreiterrolle<br />

übernommen: für das vertiefte Studium<br />

<strong>de</strong>r Mathematik für ein Lehramt an Gymnasien<br />

wird bereits ab <strong>de</strong>m Wintersemester<br />

2009/10 ein innovatives Konzept umgesetzt.<br />

Das zunächst für die Fächerkombination mit<br />

Informatik o<strong>de</strong>r Musik vorgesehene Programm<br />

wird auch schon von Studieren<strong>de</strong>n mit an<strong>de</strong>ren<br />

Zweitfächern in großer Zahl angenommen.<br />

Die Einführungsveranstaltung „Mathematik<br />

I (Lehramt Gymnasium)“ wird <strong>de</strong>rzeit von<br />

Herrn Dr. Ralf Gerkmann gelesen, <strong>de</strong>r seit <strong>de</strong>m<br />

1. Oktober 2009 als Aka<strong>de</strong>mischer Rat im<br />

Bereich <strong>de</strong>r Lehramtsausbildung tätig ist.<br />

Die voraussichtlich zum Wintersemester<br />

2010/11 erfolgen<strong>de</strong> Modularisierung auch<br />

<strong>de</strong>s Unterrichtsfachs Mathematik für ein Lehramt<br />

an Grund-, Haupt- und Realschulen wird<br />

momentan vom Mathematischen <strong>Institut</strong> mit<br />

Nachdruck vorangetrieben; das seit langem<br />

erfolgreiche Konzept eigener, speziell auf die<br />

Bedürfnisse dieser Studieren<strong>de</strong>ngruppe zugeschnittener<br />

Lehrveranstaltungen soll dabei<br />

weiterverfolgt wer<strong>de</strong>n. In diesem Zusammenhang<br />

sei auf das in <strong>de</strong>n letzten Jahren <strong>de</strong>ut-<br />

500<br />

450<br />

3<br />

400<br />

1<br />

3<br />

122<br />

134<br />

159<br />

350<br />

300<br />

250<br />

200<br />

15<br />

11<br />

33<br />

64<br />

14<br />

54<br />

87<br />

67<br />

100<br />

110<br />

107<br />

134<br />

149<br />

249<br />

224<br />

Internationaler Masterstudiengang<br />

Mathematik als Unterrichtsfach<br />

Lehramt an Gymnasien<br />

150<br />

100<br />

50<br />

0<br />

35<br />

31<br />

67<br />

37<br />

32<br />

3<br />

25<br />

35<br />

46<br />

26<br />

38<br />

35<br />

43<br />

57<br />

57<br />

85<br />

74<br />

102<br />

99<br />

125<br />

119<br />

87<br />

82 77 79<br />

167<br />

1<br />

62<br />

1 3<br />

82 79<br />

Diplom Wirtschafts<strong>mathe</strong>matik<br />

Diplom Mathematik<br />

Bachelor Mathematik


4<br />

Berichte aus <strong>de</strong>m Mathematischen <strong>Institut</strong><br />

lich gestiegene Interesse <strong>de</strong>r Studieren<strong>de</strong>n<br />

<strong>de</strong>r Wirtschaftspädagogik am Fach Mathematik<br />

hingewiesen; eine zunehmen<strong>de</strong> Zahl<br />

an künftigen Han<strong>de</strong>lslehrerinnen und Han<strong>de</strong>lslehrern<br />

besucht im Rahmen <strong>de</strong>s Hauptstudiums<br />

die für das Unterrichtsfach Mathematik<br />

angebotenen Lehrveranstaltungen und<br />

legt <strong>de</strong>n fachwissenschaftlichen Komponenten<br />

<strong>de</strong>s Ersten Staatsexamens entsprechen<strong>de</strong><br />

Prüfungen ab.<br />

Personalien Frau Prof. Dr. Francesca Biagini<br />

hat <strong>de</strong>n Ruf auf die W3-Professur in Finanz<strong>mathe</strong>matik<br />

(Nachfolge Filipović) angenommen,<br />

so dass <strong>de</strong>r sowohl für <strong>de</strong>n auslaufen<strong>de</strong>n<br />

Diplomstudiengang als auch für <strong>de</strong>n<br />

neu konzipierten Bachelorstudiengang Wirtschafts<strong>mathe</strong>matik<br />

zentrale Lehrstuhl erfolgreich<br />

wie<strong>de</strong>rbesetzt ist.<br />

Frau Prof. Dr. Kristina Reiss hat zum Wintersemester<br />

2009/10 <strong>de</strong>n Ruf auf <strong>de</strong>n Heinz<br />

Nixdorf-Stiftungslehrstuhl für Didaktik <strong>de</strong>r<br />

Mathematik an <strong>de</strong>r School of Education <strong>de</strong>r<br />

TU München angenommen; das <strong>Institut</strong> hat<br />

bereits <strong>de</strong>n Antrag auf Wie<strong>de</strong>rzuweisung <strong>de</strong>s<br />

Lehrstuhls für Didaktik <strong>de</strong>r Mathematik bei<br />

<strong>de</strong>r Hochschulleitung gestellt und sieht einer<br />

baldigen erfolgreichen Wie<strong>de</strong>rbesetzung, die<br />

für alle Lehramtsstudiengänge unverzichtbar<br />

ist, mit Zuversicht entgegen.<br />

Herr Prof. Dr. Ulrich Derenthal (Freiburg) hat<br />

einen Ruf auf eine W2-Professur in <strong>de</strong>r Algebraischen<br />

Geometrie angenommen.<br />

Veranstaltungen Auch 2009 fand in <strong>de</strong>r<br />

letzten Woche <strong>de</strong>r bayerischen Sommerferien<br />

das überaus erfolgreiche Probestudium<br />

„LMU-Mathe-Sommer“ statt. Herr Prof. Dr.<br />

Daniel Rost, unterstützt von Herrn Dr. Andreas<br />

Fackler und zahlreichen weiteren Mitarbeiterinnen<br />

und Mitarbeitern, gewährte<br />

diesmal unter <strong>de</strong>m Motto „Einfach komplex“<br />

interessierten Oberstufenschülerinnen und<br />

Oberstufenschülern einen Blick über <strong>de</strong>n<br />

reellen Tellerrand. Rund 120 Teilnehmerinnen<br />

und Teilnehmer nutzten die Gelegenheit,<br />

sich ein authentisches Bild vom Mathematikstudium<br />

an <strong>de</strong>r LMU zu machen; ein<br />

Rückblick mit Bil<strong>de</strong>rgalerie fin<strong>de</strong>t sich auf<br />

Seite 20.<br />

Die Verabschiedung <strong>de</strong>r Studieren<strong>de</strong>n, die<br />

im vergangenen aka<strong>de</strong>mischen Jahr einen<br />

Abschluss in Mathematik o<strong>de</strong>r Wirtschafts<strong>mathe</strong>matik<br />

erworben haben, erfolgt seit vergangenem<br />

Jahr in einer eigenen Veranstaltung,<br />

um unseren Ehemaligen einen feierlichen<br />

Rahmen zur Würdigung ihrer aka<strong>de</strong>mischen<br />

Leistungen zu bieten. Die erste Absolventenfeier<br />

<strong>de</strong>s Mathematischen <strong>Institut</strong>s<br />

fand am 21. November 2009 in Zusammenarbeit<br />

mit <strong>de</strong>m För<strong>de</strong>rverein Mathematik<br />

statt.<br />

Die Reihe „Mathematik am Samstag“, die sich<br />

seit nunmehr zehn Jahren an alle Interessierten,<br />

vor allem aber an Schülerinnen und<br />

Schüler <strong>de</strong>r gymnasialen Oberstufe richtet,<br />

wird auch in diesem Frühjahr mit drei interessanten<br />

Vorträgen fortgesetzt; das genaue Programm<br />

steht auf Seite 19.<br />

Die Gemeinsame Jahrestagung <strong>de</strong>r Deutschen<br />

Mathematiker-Vereinigung (DMV) und<br />

<strong>de</strong>r Gesellschaft für Didaktik <strong>de</strong>r Mathematik<br />

(GDM) wird in diesem Jahr an <strong>de</strong>r <strong>Ludwig</strong>-<br />

<strong>Maximilians</strong>-Universität München abgehalten;<br />

die Veranstaltungen fin<strong>de</strong>n in <strong>de</strong>r Woche<br />

vom 8. bis 12. März 2010 im Universitätshauptgebäu<strong>de</strong><br />

statt. Nähere Informationen<br />

fin<strong>de</strong>n Sie auf Seite 10 und im Internet unter<br />

www.math2010.<strong>de</strong>/CMS/<br />

Erwin Schörner


Als Werkstu<strong>de</strong>nt bei <strong>de</strong>r SGSS<br />

5<br />

Nach einiger Zeit reiflicher Überlegung und<br />

Abschluss <strong>de</strong>r Klausurenphase entschloss ich<br />

mich, eine Bewerbung bei <strong>de</strong>r Société Générale<br />

Securities Services auf die vakante Position<br />

<strong>de</strong>s Werkstu<strong>de</strong>nten einzureichen. Die<br />

Stellenausschreibung fand ich auf <strong>de</strong>r LMU-<br />

Seite, und <strong>de</strong>r angegebene Tätigkeitsbereich<br />

sprach mich sofort an, zumal explizit nach<br />

Wirtschafts<strong>mathe</strong>matikern gesucht wur<strong>de</strong>.<br />

Nach einigen Tagen kam endlich die Antwort:<br />

Ich wur<strong>de</strong> zum Bewerbungsgespräch<br />

eingela<strong>de</strong>n! Vor <strong>de</strong>m Gespräch begann ich<br />

Informationen zu sammeln und mir mögliche<br />

Fragen, die mir gestellt wer<strong>de</strong>n könnten,<br />

zu überlegen. Je näher das Gespräch jedoch<br />

rückte, <strong>de</strong>sto unsicherer wur<strong>de</strong> ich:<br />

Hast du dir alles überlegt Was sollst du<br />

anziehen Eher zu früh kommen, o<strong>de</strong>r genau<br />

pünktlich Anzug mit Krawatte, o<strong>de</strong>r ohne<br />

Überhaupt einen Anzug<br />

Am Morgen <strong>de</strong>s Bewerbungsgesprächs rekapitulierte<br />

ich noch ein letztes Mal, was ich<br />

gerne sagen wollte und wie ich mich verhalten<br />

sollte, und begab mich dann auf <strong>de</strong>n Weg<br />

zur SGSS. Dann die letzten Schritte, bevor ich<br />

zu <strong>de</strong>m Raum gelangte. Direkt als ich hereinkam,<br />

wur<strong>de</strong> ich von vier Personen freundlich<br />

begrüßt. Es sollte sich herausstellen, dass<br />

Bewerbungsgespräche sehr viel weniger einschüchternd<br />

sind, als man erwarten sollte.<br />

Die Stimmung während <strong>de</strong>s Bewerbungsgesprächs<br />

war recht locker und die Fragen, die<br />

gestellt wur<strong>de</strong>n, bezogen sich größtenteils auf<br />

<strong>de</strong>n eingereichten Lebenslauf. So wur<strong>de</strong> nach<br />

beson<strong>de</strong>ren Ereignissen und <strong>de</strong>ren Auswirkungen,<br />

<strong>de</strong>r Persönlichkeit und natürlich nach<br />

<strong>de</strong>n eigenen Schwächen gefragt. Je<strong>de</strong>nfalls<br />

muss ich mich wohl bei <strong>de</strong>r Klei<strong>de</strong>rwahl richtig<br />

entschie<strong>de</strong>n haben – das muss es wohl<br />

gewesen sein –, <strong>de</strong>nn schon kurz nach <strong>de</strong>m<br />

Bewerbungsgespräch stand fest, dass ich <strong>de</strong>n<br />

Job wohl antreten dürfte, und so schlug man<br />

mir vor, ich könnte einige Tage später schon<br />

zu meinem ersten Arbeitstag erscheinen.<br />

Die Abteilung Client Reporting & Performance<br />

Measurement, in <strong>de</strong>r ich nun seit<strong>de</strong>m<br />

Werkstu<strong>de</strong>nt bin, hat zur Hauptaufgabe,<br />

Fondsmanagern und Kun<strong>de</strong>n Daten über ihre<br />

Fonds zur Verfügung zu stellen. Dazu erstellen<br />

und archivieren wir vielfältige Reports,<br />

die sich mit <strong>de</strong>r Performance, <strong>de</strong>m Risiko<br />

eines Fonds o<strong>de</strong>r an<strong>de</strong>ren investitionsrelevanten<br />

Details beschäftigt. Zusätzlich stehen<br />

wir Kun<strong>de</strong>n je<strong>de</strong>rzeit bei Fragen und Anfragen<br />

zur Seite, sei es, wenn sich ein Fonds<br />

an<strong>de</strong>rs als erwartet entwickelt und <strong>de</strong>r Kun<strong>de</strong><br />

<strong>de</strong>n Grund erfahren möchte o<strong>de</strong>r wenn die<br />

Benchmark eines Fonds geän<strong>de</strong>rt wer<strong>de</strong>n<br />

soll.<br />

Die Anfangsphase in <strong>de</strong>r Arbeit war interessant,<br />

aber erfor<strong>de</strong>rte auch eine Verän<strong>de</strong>rung<br />

<strong>de</strong>r gewohnten Arbeitsweise. Der Schritt von<br />

<strong>de</strong>r Universität in die Arbeitswelt ist anfangs<br />

doch ein größerer, als man glauben mag. Die<br />

Aufgaben, die man anfänglich zu tätigen hat,<br />

sind natürlich von an<strong>de</strong>rer Natur als in <strong>de</strong>r<br />

Universität, beson<strong>de</strong>rs im Vergleich zu <strong>de</strong>n<br />

konkreten Aufgabenstellungen innerhalb <strong>de</strong>s<br />

Mathematikstudiums. Es sind keine Aufgaben<br />

zu erledigen, bei <strong>de</strong>nen man teilweise stun<strong>de</strong>nlang<br />

grübelt und dann irgendwann das<br />

wohlige Gefühl hat, die Aufgabe gelöst zu<br />

haben, son<strong>de</strong>rn eher Aufgaben von <strong>de</strong>r Kategorie<br />

„Muss gemacht wer<strong>de</strong>n“. Mit <strong>de</strong>r Zeit<br />

arbeitete ich mich aber ein, auch durch die<br />

großartige Unterstützung meiner Kollegen,<br />

und die Arbeit begann immer spannen<strong>de</strong>r<br />

zu wer<strong>de</strong>n. Anfangs waren meine Hauptauf-


6<br />

gaben, <strong>de</strong>n Bürobetrieb zu organisieren und<br />

kleine kun<strong>de</strong>nnahe Aufgaben zu erledigen,<br />

wie z.B. nachzurechnen, wo <strong>de</strong>r Fehler in einer<br />

Performancezeitreihe ist. Mein Aufgabenfeld<br />

wan<strong>de</strong>lte sich mit <strong>de</strong>r Zeit jedoch dahingehend,<br />

dass ich immer mehr eigene Projekte<br />

übernehmen durfte und immer komplexere<br />

Probleme an mich übergeben wur<strong>de</strong>n. Ich<br />

wur<strong>de</strong> ein Teil <strong>de</strong>s Teams. Inzwischen liegen<br />

meine Hauptaufgaben unter an<strong>de</strong>rem in <strong>de</strong>r<br />

Entwicklung komplexer Perfomanceberichte,<br />

<strong>de</strong>r Berechnung von Performancezeitreihen<br />

und Performancemaßen. Die Mathematik<br />

spielt bei <strong>de</strong>r von mir verrichteten Arbeit eine<br />

eher sekundäre Rolle, auch wenn man grundlegen<strong>de</strong><br />

Rechnungen beherrschen muss. Teilweise<br />

wird auch Finanz<strong>mathe</strong>matik benötigt.<br />

Das Mathematikstudium ist jedoch trotz<br />

allem von großem Nutzen, <strong>de</strong>nn die in <strong>de</strong>m<br />

Studium erlernte Arbeits- und Denkweise<br />

erleichtert viele Tätigkeiten ungemein. Ich<br />

kann je<strong>de</strong>m nur empfehlen, selbst einmal die<br />

Erfahrung während <strong>de</strong>s Studiums zu machen,<br />

in einem Unternehmen zu arbeiten – nicht<br />

nur, da man viele interessante Menschen<br />

kennen lernt, auch fachlich und persönlich<br />

kann man sich dadurch weiterentwickeln.<br />

Nicolas Schmidt<br />

Praktikum bei <strong>de</strong>r Münchener Rück –<br />

ein Erfahrungsbericht<br />

Letzten Sommer habe ich mich für das Praktikum<br />

„Inflation und In<strong>de</strong>xentwicklung“ für die<br />

Dauer von acht Wochen bei <strong>de</strong>r Münchener<br />

Rück im Bereich „Special and Financial Risk“<br />

beworben. Nach einem kurzen Vorstellungs-<br />

Praktikum<br />

gespräch bekam ich glücklicherweise eine<br />

Zusage, und schon eine Woche später sollte<br />

es losgehen.<br />

Die Abteilung, in die ich gekommen bin,<br />

beschäftigt sich hauptsächlich mit <strong>de</strong>r Rückversicherung<br />

von operationellen Risiken von<br />

Finanzinstituten je<strong>de</strong>r Art, und meine Aufgabe<br />

sollte es nun sein, grob gesagt, herauszufin<strong>de</strong>n<br />

wie hoch <strong>de</strong>nn <strong>de</strong>r „durchschnittliche“<br />

Scha<strong>de</strong>n in <strong>de</strong>n nächsten Jahren aussehen<br />

wird. Hierzu sollte nun aus vielen Jahren<br />

Scha<strong>de</strong>nserfahrung<br />

eine Inflation<br />

<strong>de</strong>r Scha<strong>de</strong>nshöhe<br />

geschätzt<br />

wer<strong>de</strong>n, welche<br />

sich anschließend<br />

in die Zukunft fortschreiben lässt. Dieses ist in<br />

diesem Business von beson<strong>de</strong>rer Be<strong>de</strong>utung,<br />

da zwar die Verträge nur über ein Kalen<strong>de</strong>rjahr<br />

laufen, aber auch noch bis zu zehn Jahre<br />

später ein Scha<strong>de</strong>n in diese Vertragsperio<strong>de</strong><br />

fallen kann. Eine gute Schätzung für die<br />

Scha<strong>de</strong>nshöhe in <strong>de</strong>n kommen<strong>de</strong>n Jahren,


7<br />

welche meist bei Gericht verhan<strong>de</strong>lt wird, ist<br />

also für eine gute Preisfindung von großer<br />

Be<strong>de</strong>utung.<br />

Zu Beginn meiner Arbeit bekam ich nun eine<br />

etwa ein 40 Seiten starke Einführung in die<br />

von <strong>de</strong>r Münchener Rück entwickelte Methodik,<br />

wie so etwas zu berechnen ist. Da ich<br />

zu diesem Zeitpunkt erst das dritte Semester<br />

absolviert hatte, war ich am Anfang noch<br />

etwas verunsichert, ob ich <strong>de</strong>nn mit <strong>de</strong>m<br />

<strong>mathe</strong>matischen Anspruch dieses Verfahrens<br />

zurecht kommen wür<strong>de</strong>, doch schneller,<br />

als ich dachte, konnte ich mich in die Materie<br />

einarbeiten. Der Hauptteil <strong>de</strong>r Arbeit war<br />

jedoch das Aufbereiten großer Datenmengen,<br />

da die Daten in verschie<strong>de</strong>nsten Formaten<br />

vorlagen. Für mich war dies i<strong>de</strong>al, da<br />

ich hier erst einmal eine gewisse Vertrautheit<br />

mit Excel erlangen konnte. Bei <strong>de</strong>r eigentlichen<br />

Berechnung waren weniger die <strong>mathe</strong>matischen<br />

Verfahren im Mittelpunkt, son<strong>de</strong>rn<br />

eher die fast philosophische Frage, welche<br />

Daten tatsächlich verwen<strong>de</strong>t wer<strong>de</strong>n sollten.<br />

Denn wie gehen wir mit Schä<strong>de</strong>n um, welche<br />

zum Beispiel wegen <strong>de</strong>r Subprimekrise beson<strong>de</strong>rs<br />

hoch waren Sind alte Datensätze von<br />

1993 repräsentativ für die Zukunft Zusätzlich<br />

erschwert wird dies noch durch die Tatsache,<br />

dass die Versicherung von operationellen<br />

Risiken für Banken ein junges Geschäftsfeld<br />

ist, wo ohnehin noch nicht so viele Erfahrungswerte<br />

vorhan<strong>de</strong>n sind und gera<strong>de</strong> in<br />

Krisenzeiten übermäßig viele Großschä<strong>de</strong>n<br />

auftreten. Zusammen mit <strong>de</strong>n Kollegen haben<br />

wir es so gelöst, dass ich für verschie<strong>de</strong>nste<br />

Eingabedaten eine Inflation geschätzt habe<br />

– wohl in <strong>de</strong>r Hoffnung, dass die Ergebnisse<br />

sich nicht zu sehr unterschei<strong>de</strong>n. Unglücklicherweise<br />

war dies nicht <strong>de</strong>r Fall, sodass die<br />

Inflation auch in näherer Zukunft sich wohl<br />

nicht genauer bestimmen lässt.<br />

Trotz <strong>de</strong>s etwas enttäuschen<strong>de</strong>n Ergebnisses<br />

war es eine interessante Erfahrung, dass<br />

in Praxis ganz an<strong>de</strong>re Probleme auftreten<br />

als beim Beweisen mit Bleistift und Papier.<br />

Ergänzend möchte ich das exzellente Betreuungsverhältnis<br />

und die Wertschätzung meiner<br />

Arbeit betonen, sowie natürlich <strong>de</strong>n schönen<br />

Alpenblick vom sechsten Stock <strong>de</strong>s Büros.<br />

Des Weiteren gab es ein Praktikantenprogramm,<br />

was angefangen von Kunstführungen<br />

bis zu speziellen Workshops einen regen<br />

Austausch mit an<strong>de</strong>ren Praktikanten und Diploman<strong>de</strong>n<br />

geför<strong>de</strong>rt hat. Beson<strong>de</strong>rs erwähnenswert<br />

ist auch das sogenannte Praktikantenforum,<br />

wo Praktikanten und Diploman<strong>de</strong>n<br />

die Möglichkeit haben, ihre Arbeit vor<br />

<strong>de</strong>r Gruppe vorzustellen. Da bei <strong>de</strong>r Münchener<br />

Rück auch viele Geologen und Physiker<br />

ein Praktikum absolviert haben, war dies ein<br />

spannen<strong>de</strong>r Ausblick über <strong>de</strong>n Tellerrand.<br />

Nach En<strong>de</strong> <strong>de</strong>r acht Wochen wur<strong>de</strong> mir angeboten,<br />

während <strong>de</strong>s Semesters für zehn Stun<strong>de</strong>n<br />

pro Woche als Werkstu<strong>de</strong>nt zu arbeiten,<br />

was ich auch gerne angenommen habe.<br />

Insgesamt kann ich ein Praktikum bei <strong>de</strong>r<br />

Münchener Rück nur weiterempfehlen.<br />

Beson<strong>de</strong>rs ist mir aufgefallen, dass ein (Wirtschafts-)Mathematiker<br />

hier vielseitigere Qualitäten<br />

mitbringen muss als nur analytisches<br />

Geschick. Insbeson<strong>de</strong>re wegen <strong>de</strong>s oft internationalen<br />

Geschäftsfel<strong>de</strong>s ist nicht nur ein<br />

sehr gutes Englisch ein Muss, son<strong>de</strong>rn auch<br />

sogenannte „Soft-Skills“, da die Mathematiker<br />

oft bei Preisverhandlungen mit Kun<strong>de</strong>n, hier<br />

die Erstversicherungen, beteiligt sind.<br />

Janis Bauer


8<br />

Erfahrungen bei <strong>de</strong>r Siemens AG<br />

Mathematik und Produktion gehen Hand in<br />

Hand, so vielfältig ist die Anwendung <strong>de</strong>r<br />

Mathematik. Das zeigt sich gera<strong>de</strong> dann als<br />

eine große Herausfor<strong>de</strong>rung, wenn man sich<br />

vom theoretischen, erlernten Wissen aus zahlreichen<br />

Vorlesungen im Grund- und vor allem<br />

Hauptstudium aufmacht und dieses Wissen in<br />

einem Wirtschaftsunternehmen in Anwendungen<br />

umsetzt. Meine Erwartung dabei war<br />

es, einerseits Methodik und Zusammenhänge<br />

aus meinem Studium prägnanter zu erkennen<br />

und sie an<strong>de</strong>rseits in <strong>de</strong>r Praxis eines Unternehmens<br />

anzuwen<strong>de</strong>n.<br />

Zunächst wur<strong>de</strong> ich bei <strong>de</strong>r Analyse <strong>de</strong>s Produktionsablaufs<br />

in einem Transformatorenwerk<br />

<strong>de</strong>r Siemens AG in Nürnberg eingesetzt.<br />

Ein solcher Ablauf umfasst je<strong>de</strong>n einzelnen<br />

Produktionsschritt vom Wareneingang<br />

bis zum Warenausgang. Bei dieser Produktion<br />

geht es nicht um Serienfertigung,<br />

son<strong>de</strong>rn um Einzelfertigung. Je<strong>de</strong>r Transformator<br />

hat so sein eigenes Wesen und je<strong>de</strong>r<br />

Prozessschritt hat stets eine an<strong>de</strong>re Vorgehensweise.<br />

Der Prozess umfasst die Herstellung<br />

von Material für die weitere Produktion<br />

ebenso wie die Herstellung einer Spule<br />

und das Prüfen <strong>de</strong>s Transformators am En<strong>de</strong><br />

<strong>de</strong>s Herstellungsprozesses. Das Ziel war dann<br />

das Aufspüren und Beseitigen sogenannter<br />

„Bottlenecks“. Darunter versteht man einen<br />

Teilprozess, <strong>de</strong>r die höchste Auslastung in<br />

<strong>de</strong>r gesamten Prozesskette besitzt und damit<br />

<strong>de</strong>n Durchfluss stark begrenzt. Ein Produkt<br />

braucht somit länger zum Kun<strong>de</strong>n, als dies<br />

<strong>de</strong>r Kun<strong>de</strong> for<strong>de</strong>rt. In diesem Zusammenhang<br />

erfasste und analysierte ich unter fachkundiger<br />

Anleitung meiner Teammitglie<strong>de</strong>r wesentliche<br />

qualitätsrelevante Parameter eines Prozesses<br />

zur Herstellung von Transformatoren,<br />

die anschließend mittels statistischer Verfahren<br />

ausgewertet wur<strong>de</strong>n. Eine Erhöhung<br />

<strong>de</strong>r Ent<strong>de</strong>ckungswahrscheinlichkeit und eine<br />

Reduzierung <strong>de</strong>r Auftretenswahrscheinlichkeit<br />

eines Fehlers, <strong>de</strong>r in einem beliebigen<br />

Schritt auftreten kann, ist Ziel <strong>de</strong>s Qualitätsmanagements.<br />

Zur Anwendung kamen hierbei<br />

bekannte Metho<strong>de</strong>n wie beispielsweise<br />

die Anwendung von Histogrammen, um<br />

Verteilungen zu folgern, sowie Varianzanalysen<br />

und Auswertungen von Korrelationen.<br />

Anfangs war ich von <strong>de</strong>r Komplexität <strong>de</strong>s<br />

Produktionsprozesses eines Transformators<br />

zwar geschockt und überwältigt, aber schon<br />

bald faszinierte mich die Möglichkeit, einen<br />

solchen Prozess durch theoretische <strong>mathe</strong>matische<br />

Metho<strong>de</strong>n greifbar zu machen und<br />

diesen zu mo<strong>de</strong>llieren. Optimierungsmetho<strong>de</strong>n<br />

wie etwa die Anwendung <strong>de</strong>s Simplex-<br />

Algorithmus und die Warteschlangentheorie<br />

stellten <strong>de</strong>n Grundstock für tiefergehen<strong>de</strong><br />

Analysen. Die erstmalige praktische Erfahrung<br />

von Kenntnissen, die mir im Studium vermittelt<br />

wor<strong>de</strong>n waren, stellte somit eine überaus<br />

spannen<strong>de</strong> Erfahrung dar.<br />

Einen Anschluss an dieses Praktikum gab<br />

es für mich als Werkstu<strong>de</strong>nt und Diplomand<br />

im Bereich Corporate Technology <strong>de</strong>r Siemens<br />

AG in München-Perlach. Die Abteilung<br />

Production Processes befasst sich mit<br />

allen Aspekten <strong>de</strong>r Optimierung von Produktionsprozessen<br />

in <strong>de</strong>n weltweit verteilten<br />

Werken <strong>de</strong>s Unternehmens. Dabei ist es<br />

die Aufgabe, Metho<strong>de</strong>n fach- und zielgerecht<br />

auf einzelne Prozessschritte anzuwen<strong>de</strong>n.<br />

Dadurch können diese schlüssig mo<strong>de</strong>lliert<br />

und optimiert wer<strong>de</strong>n, und die verwen<strong>de</strong>te<br />

Methodik wird somit – meist auf <strong>de</strong>r<br />

Basis aussagefähiger und prägnanter Charts<br />

und Präsentationen – in <strong>de</strong>n zukünftigen<br />

Produktionsverfahren in <strong>de</strong>n Werken umge-


9<br />

setzt. Eine Verkürzung <strong>de</strong>r effektiven Herstellzeit<br />

eines beliebigen Produktes, die Erhöhung<br />

<strong>de</strong>r systemgerechten Auslastung <strong>de</strong>r Maschinen-,<br />

Material- und Mitarbeiterkapazität und<br />

die Senkung von Kosten <strong>de</strong>s gesamten Prozesses<br />

sind das Ziel dieser strukturierten,<br />

analytischen Vorgehensweise. Die stochastische<br />

Mathematik kommt insbeson<strong>de</strong>re dann<br />

zum Tragen, sobald volatiler Kun<strong>de</strong>nbedarf<br />

und schwanken<strong>de</strong> Wie<strong>de</strong>rbeschaffungszeiten<br />

von Material und Maschinen berücksichtigt<br />

wer<strong>de</strong>n müssen. Dabei spielen Rechnungen<br />

mit Erwartungswerten, partiell zu integrieren<strong>de</strong>n<br />

Integralen und Rechnungen mit<br />

insbeson<strong>de</strong>re diskreten und normalverteilten<br />

Verteilungen eine große Rolle. Unsicherheiten<br />

<strong>de</strong>r Kun<strong>de</strong>nbedarfe wer<strong>de</strong>n statistisch<br />

ausgewertet.<br />

Persönlich befasse ich mich seit über einem<br />

Jahr mit <strong>de</strong>m Thema „Optimaler Einsatz<br />

von Leiharbeitskräften im Produktionsumfeld“.<br />

Ergebnisse meiner Tätigkeit wer<strong>de</strong>n im<br />

Rahmen <strong>de</strong>r Dissertation meines direkten<br />

Betreuers in seiner Arbeit verwen<strong>de</strong>t. Dieses<br />

Projekt führe ich momentan im Rahmen<br />

meiner Diplomarbeit weiter. Dabei analysiere<br />

ich mit Mitteln spieltheoretischer Axiome<br />

und Gesetze – dank <strong>de</strong>r engagierten und<br />

motivieren<strong>de</strong>n Unterstützung durch Herrn<br />

Prof. Dr. Schottenloher – einen gesamten<br />

Produktionsprozess im industriellen Umfeld.<br />

Dabei stellt die Frage nach einer optimalen<br />

Strategienausrichtung <strong>de</strong>s Managements<br />

und <strong>de</strong>s Kun<strong>de</strong>n <strong>de</strong>n Kern meiner Arbeit<br />

dar. Wichtige <strong>mathe</strong>matische Sätze, die hierbei<br />

zur Anwendung kommen, sind vor allem<br />

jene Sätze von John Nash, die Nash-Gleichgewichte<br />

untersuchen und <strong>de</strong>ren Existenz<br />

zeigen, solche Sätze, die Spiele in extensiver<br />

Form und Normalform beschreiben, sowie<br />

aus <strong>de</strong>r Wahrscheinlichkeitstheorie bekannte<br />

Sätze wie <strong>de</strong>r Satz von Bayes.<br />

Corporate Technology ist die Forschungsund<br />

Entwicklungsabteilung <strong>de</strong>s Unternehmens<br />

und unterstützt mit ihren theoretisch<br />

fundierten und praxisorientierten Ergebnissen<br />

alle internen Produktionsbetriebe. Viele Mitarbeiter<br />

sind unter <strong>de</strong>r Woche – in <strong>de</strong>r Regel<br />

von Montag bis Donnerstag – in Consulting-<br />

Projekten unterwegs, also direkt bei internen<br />

Kun<strong>de</strong>n wie zum Beispiel <strong>de</strong>m bereits<br />

erwähnten Transformatorenwerk tätig. Wichtige<br />

Grundlage für ein produktives Ergebnis<br />

war und ist für mich vor allem das selbständige<br />

Arbeiten. Gera<strong>de</strong> Recherchen außerhalb<br />

<strong>de</strong>s Siemens-Areals, wie etwa die intensive<br />

Suche nach Literatur in <strong>de</strong>r Bayerischen<br />

Staatsbibliothek o<strong>de</strong>r die direkte Kontaktaufnahme<br />

mit wissenschaftlichen <strong>Institut</strong>en,<br />

machen für mich die Arbeit spannend und<br />

interessant. Jedoch stan<strong>de</strong>n mir diese Berater,<br />

auch wenn sie nicht im Haus waren, stets<br />

durch Telefonate und E-Mails zu Hilfe, wenn<br />

ich Probleme hatte, die ich von selbst nicht<br />

lösen konnte. Wer allerdings glaubt, man kann<br />

auch in Zukunft in <strong>de</strong>r Firma-internen Fachbibliothek<br />

einfach stöbern o<strong>de</strong>r nachschlagen,<br />

<strong>de</strong>r täuscht sich. Diese Bücher lassen<br />

sich seit Neuestem nur noch online bestellen.<br />

Da mich das Arbeiten an <strong>de</strong>r Schnittstelle<br />

zwischen theoretischer Forschung und praktischer<br />

Anwendung sehr interessiert – fast<br />

an<strong>de</strong>rthalb Jahre bin ich nun durchgehend<br />

bei Siemens tätig, als Praktikant, Werkstu<strong>de</strong>nt<br />

beziehungsweise Diplomand –, ist es mein<br />

Ziel, im Anschluss an mein Mathematikstudium<br />

weiter in diesem Bereich tätig zu sein.<br />

Andreas Reichl


10<br />

Gemeinsame Jahrestagung <strong>de</strong>r Deutschen<br />

Mathematiker-Vereinigung und <strong>de</strong>r Gesellschaft<br />

für Didaktik <strong>de</strong>r Mathematik<br />

<strong>Ludwig</strong>-<strong>Maximilians</strong>-Universität München,<br />

8.-12. März 2010<br />

Zum zweiten Mal in ihrer Geschichte gestalten<br />

die Deutsche Mathematiker-Vereinigung (DMV)<br />

und die Gesellschaft für Didaktik <strong>de</strong>r Mathematik<br />

(GDM) eine gemeinsame Jahrestagung. München<br />

ist nach 1998 wie<strong>de</strong>r einmal Tagungsort<br />

<strong>de</strong>r GDM und seit 1893 auch bereits das fünfte<br />

Mal Tagungsort <strong>de</strong>r DMV. Die Organisatoren <strong>de</strong>r<br />

Tagung aus <strong>de</strong>n Reihen <strong>de</strong>r bei<strong>de</strong>n Mathematischen <strong>Institut</strong>e <strong>de</strong>r <strong>Ludwig</strong>-<strong>Maximilians</strong>-Universität<br />

und <strong>de</strong>r Technischen Universität München sowie <strong>de</strong>m Heinz Nixdorf-Stiftungslehrstuhl für Didaktik<br />

<strong>de</strong>r Mathematik an <strong>de</strong>r TUM School of Education wollen an diese Tradition anknüpfen und<br />

la<strong>de</strong>n Wissenschaftlerinnen und Wissenschaftler, Lehrkräfte und Interessierte bei<strong>de</strong>r Communities<br />

herzlich nach München ein. Ort <strong>de</strong>r Tagung wird das zentral gelegene Hauptgebäu<strong>de</strong> <strong>de</strong>r <strong>Ludwig</strong>-<br />

<strong>Maximilians</strong>-Universität sein.<br />

Die Mathematik und die Mathematikdidaktik sind eng verbun<strong>de</strong>ne Disziplinen. Die Fachdidaktik<br />

ist als Wissenschaft vom <strong>mathe</strong>matischen Denken und <strong>de</strong>ssen Vermittlung auf immer neue Impulse<br />

aus aktuellen Zweigen <strong>de</strong>r Mathematik angewiesen. An<strong>de</strong>rerseits ist <strong>de</strong>r Bezug zur Fachdidaktik<br />

auch für Fachwissenschaftler von Interesse und das ganz beson<strong>de</strong>rs im Hinblick auf die Ausbildung<br />

<strong>de</strong>r Studieren<strong>de</strong>n und <strong>de</strong>s wissenschaftlichen Nachwuchses. Eine gemeinsame Jahrestagung<br />

stellt eine herausragen<strong>de</strong> Möglichkeit dar, diesen gegenseitigen Austausch zu nutzen und<br />

zu intensivieren.<br />

Das Programm <strong>de</strong>r Tagung wird die traditionellen Teile <strong>de</strong>r Jahrestagungen von DMV und GDM<br />

in <strong>de</strong>r gewohnten Form berücksichtigen. Auch wenn diese teilweise parallel geplant sind, sind<br />

doch Mitglie<strong>de</strong>r <strong>de</strong>r jeweils an<strong>de</strong>ren Community herzlich eingela<strong>de</strong>n. Als Hauptvortragen<strong>de</strong> im<br />

Bereich <strong>de</strong>r Mathematik konnten gewonnen wer<strong>de</strong>n:<br />

••<br />

Folkmar Bornemann (TU München)<br />

••<br />

Jonathan Borwein (University of Newcastle)<br />

••<br />

Anton Bovier (Universität Bonn)<br />

••<br />

Leonid Polterovich (University of Chicago & MSRI Berkley)<br />

••<br />

Tristan Rivière (ETH Zürich)<br />

••<br />

Katrin Tent (Universität Münster)<br />

Weiterhin wird Annette Werner (Johann Wolfgang Goethe-Universität Frankfurt) die Emmy<br />

Noether Lecture halten. Für die Hauptvorträge zur Fachdidaktik, die parallel zu <strong>de</strong>n Vormittagsvorträgen<br />

<strong>de</strong>r Mathematik stattfin<strong>de</strong>n, haben die folgen<strong>de</strong>n Kolleginnen und Kollegen zugesagt:<br />

••<br />

Elisabeth Moser Opitz (Universität Zürich)<br />

••<br />

Lieven Verschaffel (Katholieke Universiteit Leuven)<br />

••<br />

Aiso Heinze (Leibniz-<strong>Institut</strong> für die Pädagogik <strong>de</strong>r Naturwissenschaften Kiel)<br />

••<br />

Philipp Mayring (Universität Klagenfurt)<br />

Über die Hauptvorträge hinaus sind natürlich Einzelbeiträge vorgesehen, die in thematischen


11<br />

Sektionen und – im Bereich <strong>de</strong>r Mathematik – in Minisymposien organisiert wer<strong>de</strong>n. Sowohl<br />

Einzelbeiträge als auch Minisymposien bzw. Beiträge darin können bis spätestens 31. Januar über<br />

die Tagungshomepage www.math2010.<strong>de</strong> angemel<strong>de</strong>t wer<strong>de</strong>n.<br />

Um beson<strong>de</strong>rs Gemeinsamkeiten <strong>de</strong>r bei<strong>de</strong>n Forschungsfel<strong>de</strong>r zu betonen sind, wie schon<br />

auf <strong>de</strong>r letzten gemeinsamen Tagung in Berlin, beson<strong>de</strong>re Vorträge vorgesehen, die an <strong>de</strong>r<br />

Schnittstelle von Fach und Fachdidaktik angesie<strong>de</strong>lt sind und potentiell Interessen von<br />

Teilnehmern bei<strong>de</strong>r Bereiche ansprechen:<br />

••<br />

Persi Diaconis (Stanford University)<br />

••<br />

Hans-Niels Jahnke (Universität Duisburg-Essen)<br />

••<br />

Ueli Maurer (ETH Zürich)<br />

••<br />

Francesca Biagini (LMU München)<br />

Auch <strong>de</strong>r Eröffnungsvortrag <strong>de</strong>r Tagung wur<strong>de</strong> bewusst an <strong>de</strong>r Schnittstelle angesie<strong>de</strong>lt. Dafür<br />

konnte Jürgen Baumert (Max-Planck-<strong>Institut</strong> für Bildungsforschung, Berlin) gewonnen wer<strong>de</strong>n.<br />

Darüber hinaus wer<strong>de</strong>n gemeinsam mit <strong>de</strong>n bei<strong>de</strong>n Fachgesellschaften DMV und GDM weitere<br />

Schnittstellenaktivitäten organisiert, die sich fokussiert mit Themen befassen, zu <strong>de</strong>nen Expertise<br />

bei<strong>de</strong>r Communities notwendig ist o<strong>de</strong>r die für bei<strong>de</strong> Seiten von wechselseitigem Interesse sind.<br />

Am Dienstag <strong>de</strong>n 9. März wer<strong>de</strong>n Veranstaltungen und Vorträge gebün<strong>de</strong>lt, die beson<strong>de</strong>rs auch<br />

für aktive Lehrkräfte aller Schularten von Interesse sind. Die Teilnahme an <strong>de</strong>r Tagung nur an<br />

diesem einen Tag ist daher für alle Lehrkräfte möglich.<br />

Neben <strong>de</strong>n wissenschaftlichen Aktivitäten gibt ein vielfältiges Rahmenprogramm Möglichkeiten<br />

zum fachlichen und überfachlichen Austausch. So können Karten für Oper und Theater über die<br />

Tagungshomepage gebucht wer<strong>de</strong>n. Für <strong>de</strong>n Mittwoch Nachmittag sind Angebote vorgesehen,<br />

die zum Kennlernen <strong>de</strong>r bayerischen Lan<strong>de</strong>shauptstadt München einla<strong>de</strong>n und ihre bekannten<br />

wie unbekannten Seiten zeigen sollen.<br />

Wir wür<strong>de</strong>n uns sehr freuen, Sie im März in München begrüßen zu dürfen und wünschen Ihnen<br />

und uns eine produktive, abwechslungsreiche Tagung und einen intensiven Austausch.<br />

Kristina Reiss (Heinz Nixdorf-Stiftungslehrstuhl für Didaktik <strong>de</strong>r Mathematik, TUM School of<br />

Education, TU München)<br />

Gregor Kemper (Dekan <strong>de</strong>s Zentrums Mathematik <strong>de</strong>r TU München)<br />

Detlef Dürr (Direktor <strong>de</strong>s Mathematischen <strong>Institut</strong>s <strong>de</strong>r LMU München)<br />

für das lokale Organisationskomitee<br />

Das Wichtigste im Überblick<br />

Termin 8.-12. März 2010<br />

Lehrertag 9. März 2010<br />

Ort<br />

München<br />

Hauptgebäu<strong>de</strong> <strong>de</strong>r LMU am Geschwister-Scholl-Platz<br />

Anmeldung Vergünstigt bis 6.1.2010<br />

Beitragseinreichung Bis spätestens 31.1.2010<br />

Veranstalter<br />

Weitere Informationen, Anmeldung,<br />

Beitragseinreichung, Hotelbuchung<br />

<strong>Mathematisches</strong> <strong>Institut</strong> <strong>de</strong>r LMU München<br />

Heinz Nixdorf-Stiftungslehrstuhl,<br />

TUM School of Education, TU München<br />

Zentrum Mathematik <strong>de</strong>r TU München<br />

Tagungshomepage www.math2010.<strong>de</strong>


Deine Facharbeit<br />

in <strong>de</strong>r 1. Liga!<br />

Hol dir <strong>de</strong>n Dr. Hans Riegel-Fachpreis<br />

für die beste Facharbeit in<br />

Erdkun<strong>de</strong> o<strong>de</strong>r Mathematik!<br />

Zu gewinnen gibt es in je<strong>de</strong>m Fach:<br />

1. Preis: € 600<br />

2. Preis: € 400<br />

3. Preis: € 200<br />

Nähere Informationen<br />

fin<strong>de</strong>st du hier:<br />

www.math.<strong>lmu</strong>.<strong>de</strong>/fachpreise<br />

Du kannst dich bewerben,<br />

falls du <strong>de</strong>ine Facharbeit in<br />

Erdkun<strong>de</strong> o<strong>de</strong>r Mathematik<br />

im Schuljahr 2009/2010<br />

geschrieben hast und<br />

diese <strong>de</strong>iner Meinung nach<br />

ausgezeichnet gelungen ist.<br />

Einsen<strong>de</strong>schluss: 1. Mai 2010<br />

Deine Facharbeit wird<br />

von Fachwissenschaftlern<br />

<strong>de</strong>r <strong>Ludwig</strong>-<strong>Maximilians</strong>-<br />

Universität München<br />

begutachtet.<br />

Vergeben von <strong>de</strong>r<br />

<strong>Ludwig</strong>-<strong>Maximilians</strong>-Universität<br />

München und <strong>de</strong>r<br />

Dr. Hans Riegel-Stiftung.


13<br />

Das Traineeprogramm <strong>de</strong>r BayernLB<br />

Learning by Banking<br />

Die BayernLB ist ein starker und verlässlicher Finanzpartner – verwurzelt in Bayern, geprägt<br />

durch enge Kun<strong>de</strong>nbeziehungen und ausgezeichnet durch individuelle Finanzlösungen für<br />

unsere Kun<strong>de</strong>n sowie <strong>de</strong>ren nationale und internationale Marktaktivitäten.<br />

Sie haben einen überdurchschnittlichen Abschluss in Wirtschaftswissenschaften o<strong>de</strong>r Jura<br />

und bringen erste Praxiserfahrung im Finanzwesen mit Sie sind engagiert und haben Spaß<br />

an <strong>de</strong>r Dienst leistung Dann haben Sie beste Voraus setzungen für die Aufnahme in unser<br />

Traineeprogramm. 15 Monate lang arbeiten Sie in einer international tätigen Großbank.<br />

In einem maßgeschnei<strong>de</strong>rten Programm wer<strong>de</strong>n Sie dabei intensiv und individuell von uns<br />

geför<strong>de</strong>rt – nach Ihren Fähigkeiten und nach Ihren Neigungen. Ihr Gewinn: Professionalität<br />

und eine faszinie ren<strong>de</strong> Berufsperspektive in <strong>de</strong>r Welt <strong>de</strong>r Wirtschaft.<br />

Interessiert Dann richten Sie Ihre Bewerbung an:<br />

Bayerische Lan<strong>de</strong>sbank (BayernLB)<br />

Corporate Center Bereich Personal<br />

Personalentwicklung, 1630<br />

Nachwuchsentwicklung<br />

80277 München<br />

Telefon 089 2171-24915<br />

trainee@bayernlb.<strong>de</strong> . www.bayernlb.<strong>de</strong>


14<br />

Karrieren<br />

Ein Münchner<br />

Mathematiker<br />

in Wales an <strong>de</strong>r<br />

Prifysgol<br />

Abertawe<br />

Während meines Mathematikstudiums<br />

an <strong>de</strong>r LMU<br />

München fühlte ich mich am meisten zur<br />

Topologie und zur Logik hingezogen. Bei<strong>de</strong><br />

Disziplinen sind gute Beispiele dafür, wie<br />

man in <strong>de</strong>r Mathematik durch Abstraktion<br />

und Einnahme eines höheren Standpunkts<br />

Theorien von gera<strong>de</strong>zu schwin<strong>de</strong>lerregen<strong>de</strong>r<br />

Allgemeinheit und Vereinheitlichungskraft<br />

gewinnen kann: in <strong>de</strong>r Topologie (die mir<br />

damals in exzellenter Weise von Prof. Roelcke<br />

vermittelt wur<strong>de</strong>) die einheitliche Behandlung<br />

verschie<strong>de</strong>nster Konvergenz- und Stetigkeitsbegriffe,<br />

in <strong>de</strong>r Logik ein überraschend<br />

einfacher Herleitungskalkül, <strong>de</strong>r in <strong>de</strong>r Lage<br />

ist, alle logisch gültigen Aussagen formal zu<br />

beweisen (Gö<strong>de</strong>ls Vollständigkeitssatz), sowie<br />

die Zermelo-Fraenkelsche Mengenlehre, in<br />

<strong>de</strong>r die gesamte (klassische) Mathematik formalisiert<br />

wer<strong>de</strong>n kann. Darüber hinaus zeigt<br />

die Logik prinzipiell unüberwindbare Grenzen<br />

<strong>de</strong>r Mathematik auf: Nach <strong>de</strong>m Ersten<br />

Unvollständigkeitssatz von Gö<strong>de</strong>l gibt es in<br />

je<strong>de</strong>m wi<strong>de</strong>rspruchsfreien formalen System<br />

<strong>de</strong>r Mathematik, welches ein Minimum an<br />

Zahlentheorie umfasst, Aussagen, die in ihr<br />

nicht entscheidbar, also we<strong>de</strong>r beweisbar<br />

noch wi<strong>de</strong>rlegbar sind. Aber es kommt noch<br />

„schlimmer“: Es existieren nicht nur irgendwelche<br />

verqueren unentscheidbare Aussagen,<br />

<strong>de</strong>ren Wahrheit nieman<strong>de</strong>n interessiert,<br />

nein, Gö<strong>de</strong>l zeigte in seinem Zweiten Unvollständigkeitssatz,<br />

dass ausgerechnet<br />

die gemäß „Hilbert's<br />

Programm“ wichtigste<br />

Aussage, nämlich die Wi<strong>de</strong>rspruchsfreiheit<br />

<strong>de</strong>s betrachteten<br />

Systems, in ebendiesem<br />

nicht beweisbar ist.<br />

Genauso, wie ich mich jetzt<br />

dazu habe hinreißen lassen,<br />

<strong>de</strong>n Leser mit einigen <strong>de</strong>r<br />

„highlights“ <strong>de</strong>r Logik zu „überfallen“ – wo<br />

ich doch über meinen Wer<strong>de</strong>gang nach <strong>de</strong>m<br />

Studium berichten will –, war ich damals von<br />

<strong>de</strong>r Logik hingerissen. Meine Doktorarbeit<br />

(1990) beschäftigte sich mit einer faszinieren<strong>de</strong>n<br />

Verbindung von Topologie, Berechenbarkeit<br />

und Logik (Domain Theory). Auf einem<br />

von mir entwickelten topologischen Begriff<br />

<strong>de</strong>r Totalität aufbauend konnte ich zeigen,<br />

dass unter sehr allgemeinen Bedingungen<br />

berechenbare totale Funktionale automatisch<br />

stetig sind. In meiner Habilitationsschrift<br />

(1997) verallgemeinerte ich diese Ergebnisse<br />

auf Mo<strong>de</strong>lle <strong>de</strong>r intuitionistischen Typentheorie.<br />

Da die in <strong>de</strong>r Münchner Logikgruppe betriebene<br />

Forschung zahlreiche Verbindungen<br />

zur Informatik hat, war es nur natürlich, mich<br />

nach einer Stelle in diesem Gebiet umzusehen.<br />

Dass ich dabei allerdings in Wales (auf<br />

walisisch „Cymru“, ausgesprochen „Kamri“)<br />

lan<strong>de</strong>n wür<strong>de</strong>, hätte ich mir nicht träumen<br />

lassen.<br />

Als ich 1999 meine Stelle als Lecturer an<br />

<strong>de</strong>r Prifysgol Abertawe (Universität Swansea)<br />

antrat – inzwischen bin ich Rea<strong>de</strong>r –,<br />

war das dortige Computer Science Department<br />

sehr klein. Die Theoretische Informatik


15<br />

in Swansea umfasste damals, mit mir als Neuzugang<br />

und einzigem Auslän<strong>de</strong>r, vier Dozenten.<br />

Inzwischen hat sich ihre Größe fast verdreifacht,<br />

und sie besteht nun gut zur Hälfte<br />

aus <strong>de</strong>utschen Logikern, darunter drei von<br />

<strong>de</strong>r LMU. Die von <strong>de</strong>r Gruppe vertretene Forschung<br />

umfasst z.B. Algebraische Spezifikation,<br />

Beweistheoretische Metho<strong>de</strong>n in <strong>de</strong>r<br />

Komplexitätstheorie und Programmextraktion,<br />

Prozess-Algebra, Semantik von Programmiersprachen,<br />

aber auch Angewandtes wie<br />

die Verifikation elektronischer Schaltungsanlagen<br />

auf britischen Bahnhöfen – mein Kollege<br />

Anton Setzer, einer <strong>de</strong>r drei Münchner<br />

Logiker, ist an letzterem Projekt beteiligt.<br />

Meine eigene aktuelle Forschung befasst<br />

sich vorwiegend mit <strong>de</strong>r oben genannten Programmextraktion<br />

aus Beweisen und <strong>de</strong>ren<br />

Anwendung – zum Beispiel auf die exakte<br />

Berechnung reeller Funktionen.<br />

In Swansea wird aber Logik nicht nur betrieben,<br />

bei uns treffen sich auch viele Logiker<br />

aus aller Welt. Seit 2006 veranstalteten wir in<br />

Swansea je<strong>de</strong>s Jahr eine internationale Logik-<br />

Tagung. Das Gruppenphoto wur<strong>de</strong>n während<br />

eines Lunch-Picnics<br />

von „Proof, Computation,<br />

Complexity“<br />

(April 2007) am<br />

Strand vor <strong>de</strong>r Uni aufgenommen.<br />

Mancher<br />

Leser wird darauf einige<br />

ehemalige o<strong>de</strong>r jetzige<br />

„Münchner“ erkennen.<br />

das Maß an Respekt, mit <strong>de</strong>m man behan<strong>de</strong>lt<br />

wird, wenn man aus <strong>de</strong>r Münchner Logikschule<br />

hervorgegangen ist. Die britischen<br />

Logiker wissen erstaunlich gut Bescheid über<br />

die Logik in München. Die aktuellen Entwicklungen<br />

dort wer<strong>de</strong>n hier in Großbritannien<br />

mit großem Interesse verfolgt.<br />

Jetzt lebe ich seit zehn Jahren mit meinen<br />

bei<strong>de</strong>n Söhnen und meiner Frau Monika Seisenberger<br />

– auch eine Münchner Logikerin<br />

und Kollegin – in Swansea. Wir genießen die<br />

Weite <strong>de</strong>s Meeres und die wil<strong>de</strong> Küstenlandschaft<br />

<strong>de</strong>s Gowers (eine an Swansea angrenzen<strong>de</strong><br />

Halbinsel) – was nicht heißt, dass wir<br />

unsere bayerische Heimat und die Berge nicht<br />

vermissen wür<strong>de</strong>n.<br />

Mit beson<strong>de</strong>rer Freu<strong>de</strong> und Stolz erfüllt mich,<br />

dass <strong>de</strong>r rege wissenschaftliche Austausch<br />

mit meinem Doktorvater Prof. Schwichtenberg<br />

und seiner Münchner Logikgruppe nach<br />

wie vor besteht. Ich hoffe, dass das noch<br />

lange so bleiben wird.<br />

Ulrich Berger<br />

Was mich am meisten<br />

überraschte, als ich die<br />

britische „Logic Communit<br />

y “ nach und<br />

nach kennenlernte, war


16<br />

Karrieren<br />

Mathematik<br />

unterrichten,<br />

wo viele gerne<br />

Urlaub machen<br />

South Beach, Evergla<strong>de</strong>s, Florida<br />

Keys, Cape Canaveral,<br />

Disney World – beim Stichwort<br />

Südflorida <strong>de</strong>nkt vielleicht<br />

nicht je<strong>de</strong>r gleich an<br />

die Mathematik. Wie es mich in die Region<br />

verschlagen hat, und warum die Arbeit hier<br />

durchaus ihre Reize hat, möchte ich in diesem<br />

Bericht darlegen.<br />

Lehrjahre<br />

Studiert und promoviert habe ich an <strong>de</strong>r Uni<br />

München. Was war an meinem Studium rückblickend<br />

beson<strong>de</strong>rs wichtig Zuerst natürlich<br />

die ausgezeichneten Algebraveranstaltungen,<br />

die ich bei <strong>de</strong>n Professoren Zimmermann<br />

– meinem späteren Doktorvater –, Zöschinger<br />

und Schnei<strong>de</strong>r besuchen konnte. Und<br />

dann die Ermutigung, Projekte von beson<strong>de</strong>rem<br />

Interesse mit Entschlossenheit zu verfolgen;<br />

dazu möchte ich drei Beispiele geben:<br />

In <strong>de</strong>r Darstellungstheorie haben es die<br />

neuen Metho<strong>de</strong>n von Auslan<strong>de</strong>r und Reiten<br />

möglich gemacht, unter Verwendung von<br />

Kombinatorik, Ringtheorie und Homologischer<br />

Algebra Kategorien von Moduln ganz<br />

genau zu beschreiben. (Ein Beispiel einer solchen<br />

Kategorie ist auf <strong>de</strong>m Photo mit <strong>de</strong>n<br />

Kaffeebechern für einen Schüler<strong>mathe</strong>wettbewerb.)<br />

Die Darstellungstheorie wur<strong>de</strong> also<br />

mein Diplom- und Promotionsthema.<br />

Während in <strong>de</strong>r Stadt München<br />

damals Bürgerinitiativen<br />

zur Verkehrspolitik zunehmend<br />

aktiv wur<strong>de</strong>n, konnte<br />

ich mich an <strong>de</strong>r Uni an <strong>de</strong>r<br />

Gründung <strong>de</strong>s Arbeitskreises<br />

Rad <strong>de</strong>r Fachschaft Mathematik<br />

beteiligen. Wir haben<br />

Diskussionsveranstaltungen,<br />

Aktionstage und die drei<br />

Münchner Fahrrad<strong>de</strong>mos<br />

1990–92 organisiert. (Und<br />

wur<strong>de</strong>n dafür auch einmal<br />

nach Bonn eingela<strong>de</strong>n!)<br />

Zurück zur Mathematik. Mein erster Uniabschluss<br />

ist übrigens ein Master of Science,<br />

<strong>de</strong>nn ich konnte mit einem Stipendium <strong>de</strong>s<br />

DAAD ein Jahr lang an <strong>de</strong>r Uni Warwick in<br />

England studieren. Ich habe mich dort entschlossen,<br />

eine Promotion in <strong>de</strong>r Mathematik<br />

anzustreben.<br />

Wan<strong>de</strong>rjahre<br />

Dagegen habe ich mich nie bewusst für eine<br />

Unikarriere entschie<strong>de</strong>n. Als sich jedoch<br />

nach <strong>de</strong>r Promotion 1996 die Möglichkeit<br />

ergab, für zwei Jahre als wissenschaftlicher<br />

Assistent an <strong>de</strong>r Karlsuniversität Prag zu<br />

arbeiten, konnte ich nicht wi<strong>de</strong>rstehen. Dort<br />

habe ich Übungen betreut, durfte aber auch<br />

(an <strong>de</strong>r ältesten Universität Mitteleuropas!)<br />

eigene Veranstaltungen anbieten, etwa „Darstellungstheorie<br />

von Gruppen“ o<strong>de</strong>r „Knotentheorie<br />

und Quantengruppen“.<br />

Nach Prag war ich für ein Jahr als Postdoktorand<br />

in Antwerpen, unterstützt durch ein<br />

TMR-Stipendium <strong>de</strong>r Europäischen Union.<br />

Dann ergab sich die Möglichkeit, als Visiting<br />

Professor für ein Jahr in <strong>de</strong>n Vereinigten Staa-


17<br />

ten zu arbeiten. An dieser Stelle möchte ich<br />

mich bei Professor Zöschinger und Dr. Schuster<br />

für die Vermittlung bedanken.<br />

Aus <strong>de</strong>r Neuen Welt<br />

Aus <strong>de</strong>m Jahr als Besucher wur<strong>de</strong>n zwei, und<br />

als ich mich dann auf Stellen bewarb, habe<br />

ich das Angebot einer Assistenzprofessur an<br />

<strong>de</strong>r Florida Atlantic University gerne angenommen.<br />

Die Staatsuniversität versorgt mit<br />

acht „campuses“ die Ostküste von Südflorida,<br />

insbeson<strong>de</strong>re die Gegend um Fort Lau<strong>de</strong>rdale<br />

und West Palm Beach. Ich selbst lebe<br />

und arbeite in Boca Raton, einer Stadt, die<br />

durch Tennis und die Entwicklung <strong>de</strong>s IBM<br />

PC bekannt gewor<strong>de</strong>n ist, die aber auch als<br />

Spamhauptstadt <strong>de</strong>r Welt gilt. Die Stadt ist<br />

schön angelegt, so gibt es etwa keine Werbetafeln<br />

und keine Autohändler. Was ich sehr<br />

schätze, ist, dass das Zentrum nicht ganz am<br />

Meer liegt, so dass man am Strand zwischen<br />

Dünen und <strong>de</strong>m Atlantik stun<strong>de</strong>nlang wan<strong>de</strong>rn<br />

kann.<br />

Unser Department bietet Bachelor- und Master-Abschlüsse<br />

an, veranstaltet aber auch<br />

viele Servicekurse für Naturwissenschaftler,<br />

Ingenieure, sogar Geisteswissenschaftler.<br />

Wir haben ein Promotionsprogramm mit<br />

etwa 50 Doktoran<strong>de</strong>n. Gewöhnlich biete ich<br />

je<strong>de</strong>s Semester zwei drei- o<strong>de</strong>r vierstündige<br />

Veranstaltungen an, meist für Ingenieure<br />

o<strong>de</strong>r für Mathematikstu<strong>de</strong>nten im Grundstudium,<br />

gelegentlich sind auch Vorlesungen<br />

für Doktoran<strong>de</strong>n dabei. Daneben organisiere<br />

ich das Algebraseminar. Meine Arbeit<br />

an <strong>de</strong>r FAU wird nach Forschung, Lehre und<br />

Service beurteilt. Aus je<strong>de</strong>m <strong>de</strong>r drei Bereiche<br />

möchte ich einen Höhepunkt vorstellen.<br />

Das Untergruppenproblem<br />

In <strong>de</strong>r Forschung habe ich in Boca Raton<br />

begonnen, am Birkhoff Problem zur Klassifikation<br />

von Einbettungen einer Untergruppe<br />

in einer endlichen abelschen Gruppe zu<br />

arbeiten. Als ich bei einem Besuch am Son<strong>de</strong>rforschungsbereich<br />

in Bielefeld Professor<br />

Ringel davon berichtete, fand er das Problem<br />

so interessant, dass sich eine Zusammenarbeit<br />

ergab, aus <strong>de</strong>r bisher drei Arbeiten entstan<strong>de</strong>n<br />

sind.<br />

Die Ergebnisse zum Problem von Birkhoff<br />

haben unmittelbare Anwendung in <strong>de</strong>r Linearen<br />

Algebra, insbeson<strong>de</strong>re zur Klassifizierung<br />

<strong>de</strong>r invarianten Unterräume eines linearen<br />

Operators. Oft trage ich auf Tagungen o<strong>de</strong>r<br />

bei Kolloquien über das Birkhoff Problem<br />

o<strong>de</strong>r die Anwendungen vor.<br />

Meine erste Doktorandin<br />

Audrey Moore kam im Herbst 2006 an die<br />

FAU mit einem Master von <strong>de</strong>r Virginia Tech<br />

University. Sie arbeitete mit mir seit <strong>de</strong>m<br />

Sommer 2007 und wur<strong>de</strong> im Herbst meine<br />

erste Doktorandin. Während meines Forschungsfreisemesters<br />

im Herbst 2008 haben<br />

Audrey und ich <strong>de</strong>n Son<strong>de</strong>rforschungsbereich<br />

in Bielefeld besucht. Dort konnte sie insbe-


18<br />

son<strong>de</strong>re <strong>de</strong>n Satz von Auslan<strong>de</strong>r und Ringel-<br />

Tachikawa für Darstellungen eines Posets verallgemeinern,<br />

ein Ergebnis, das mich beson<strong>de</strong>rs<br />

freut, da es auf einem Beweis <strong>de</strong>s klassischen<br />

Satzes durch meinen Doktorvater aufbaut.<br />

Audrey (auf <strong>de</strong>m Bild bei einem Ausflug nach<br />

Neuschwanstein) konnte schon im Frühjahr<br />

darauf an <strong>de</strong>r University of California in Santa<br />

Barbara und dann auf <strong>de</strong>r Internationalen<br />

Auslan<strong>de</strong>r Tagung in Woods Hole in Massachusetts<br />

über ihre Arbeit vortragen. Sie verteidigte<br />

ihre Dissertation im Juni 2009 und<br />

ist jetzt als Assistenzprofessorin an <strong>de</strong>r Delaware<br />

State University tätig.<br />

Mathewettbewerbe an Highschools<br />

Im Frühjahr 2002 leitete <strong>de</strong>r Chair unseres<br />

Departments eine E-Mail einer Lehrerin weiter:<br />

Ihre Schule sucht Professoren, die bereit sind,<br />

bei einem Mathewettbewerb im „dispute<br />

center“ mitzuarbeiten – klingt das nicht nach<br />

etwas Außergewöhnlichem Zu dritt haben wir<br />

die Schule besucht: An einem wun<strong>de</strong>rschönen<br />

Samstagvormittag kamen dort über 1000<br />

Schüler von etwa 40 Highschools in Südflorida<br />

zusammen, um einen ganzen Tag lang<br />

in Einzel-, Gruppen- und Schulwettbewerben<br />

Mathematikaufgaben zu lösen!<br />

Die Schüler sind in Teams organisiert, und<br />

je<strong>de</strong>s Team hat sein eigenes T-Shirt, oft mit<br />

einem witzigen Mathemotiv. Was ist unsere<br />

Aufgabe Falls nun bei diesen vielen Wettbewerben<br />

Schüler mit <strong>de</strong>r offiziellen Lösung<br />

eines Problems nicht einverstan<strong>de</strong>n sind,<br />

können sie „disputes“ einreichen, über die<br />

wir drei dann zu entschei<strong>de</strong>n haben. Zum<br />

Abschluss wer<strong>de</strong>n in <strong>de</strong>r Turnhalle über 200<br />

„trophies“ an die Sieger verliehen: Die Schüler<br />

jubeln sich zu wie bei einem Sportereignis<br />

– was die Veranstaltung auch ist.<br />

Seit <strong>de</strong>m Tag besuche ich regelmäßig regionale<br />

Mathewettbewerbe dieser Art, und zweimal<br />

schon konnten wir ein solches Ereignis<br />

bei uns an <strong>de</strong>r Uni veranstalten<br />

(siehe Photo). Bei<br />

je<strong>de</strong>m Wettbewerb freut<br />

es mich, Lehrer zu treffen,<br />

die unglaublich viel Energie<br />

in das Training ihrer<br />

Teams investieren und<br />

damit ihren Schülern ausgezeichnete<br />

Möglichkeiten<br />

bei <strong>de</strong>r Bewerbung um<br />

einen Studienplatz verschaffen.<br />

Insgesamt haben mir die Ausbildung in München<br />

und die Unterstützung aus Bielefeld es<br />

ermöglicht, hier in Florida einen faszinieren<strong>de</strong>n<br />

Beruf auszuüben. Ich schätze beson<strong>de</strong>rs<br />

Projekte, etwa mit Lehrern bei <strong>de</strong>r Organisation<br />

von Highschool-Mathewettbewerben,<br />

genieße die Möglichkeit, Kollegen, insbeson<strong>de</strong>re<br />

in Deutschland, besuchen zu können,<br />

und freue mich über je<strong>de</strong>n Stu<strong>de</strong>nten, <strong>de</strong>r<br />

mit einem Matheproblem an meiner Bürotür<br />

anklopft.<br />

Markus Schmidmeier


Mathematik am Samstag<br />

19<br />

Samstag, <strong>de</strong>n 06.03.2010, 14.15 – 15.30<br />

Raum M 010, Hauptgebäu<strong>de</strong>, Geschwister-Scholl-Platz 1<br />

Prof. Dr. Peter Müller<br />

Über Nachtwächter und Ameisen in<br />

einem zufälligen Irrgarten<br />

Der Vortrag stellt zwei Irrfahrtsprobleme vor, zwischen <strong>de</strong>nen ein<br />

erst vor wenigen Jahren gefun<strong>de</strong>ner, überraschen<strong>de</strong>r Zusammenhang<br />

besteht. Zum einen han<strong>de</strong>lt es sich um eine Irrfahrt auf einem<br />

regelmäßigen Gitter mit einem zusätzlichen Freiheitsgrad, zum<br />

an<strong>de</strong>ren um eine einfache Irrfahrt in einer zufälligen Umgebung.<br />

Samstag, <strong>de</strong>n 20.03.2010, 14.15 – 15.30<br />

Raum A 021, Hauptgebäu<strong>de</strong>, Geschwister-Scholl-Platz 1<br />

Priv.-Doz. Dr. Andreas Hinz<br />

Der Turm von Hanoi – Graphen<br />

weisen <strong>de</strong>n rechten Weg<br />

Der Turm von Hanoi ist ein Solitärspiel, das En<strong>de</strong> <strong>de</strong>s 19. Jahrhun<strong>de</strong>rts von<br />

<strong>de</strong>m Zahlentheoretiker Edouard Lucas erfun<strong>de</strong>n wur<strong>de</strong>. Sein <strong>mathe</strong>matisches<br />

Mo<strong>de</strong>ll basiert auf Graphen, wie sie uns z.B. von U-Bahn-Plänen<br />

vertraut sind. Es führt auf Verbindungen zu altindischer Lyrik, italienischen<br />

Ornamenten und chinesischen Co<strong>de</strong>s. Praktische Anwendung fin<strong>de</strong>t <strong>de</strong>r<br />

Turm von Hanoi als Test in <strong>de</strong>r kognitiven Neuropsychologie.<br />

Samstag, <strong>de</strong>n 17.04.2010, 14.15 – 15.30<br />

Voraussichtlich: B 005, <strong>Mathematisches</strong> <strong>Institut</strong>, Theresienstr. 39<br />

Prof. Dr. Heinrich Steinlein<br />

Wo parkt man im Weltraum am<br />

billigsten<br />

Die NASA-Son<strong>de</strong> Genesis hatte die Aufgabe, winzige Mengen von<br />

Sonnenwindpartikeln einzufangen und sicher zur Er<strong>de</strong> zu bringen. Dazu<br />

war es nötig, die Raumson<strong>de</strong> außerhalb <strong>de</strong>r Erdatmosphäre und <strong>de</strong>s<br />

Erdmagnetfel<strong>de</strong>s für gut drei Jahre zu positionieren und danach sicher<br />

zur Er<strong>de</strong> zurückzuholen. Welche Parkposition bot sich für diese Mission<br />

an und was steckt an Mathematik dahinter<br />

Ein Lageplan für die Hörsäle A 021 und M 010 im Hauptgebäu<strong>de</strong><br />

ist auf Seite 31 abgedruckt.


20<br />

10 Jahre Probestudium Mathematik<br />

Vom 7. bis 11. September 2009 fand das 10.<br />

Probestudium <strong>de</strong>s Mathematischen <strong>Institut</strong>s<br />

statt; ein guter Anlass, zurückzublicken:<br />

Im September 2000 konnten knapp 100<br />

Schülerinnen und Schüler beim ersten Probestudium<br />

zum Thema „Wachstum, Schwingungen<br />

und ein Blick ins Chaos“ begrüßt wer<strong>de</strong>n.<br />

Von Herrn Dr. Schörner und Herrn Emmer<br />

konzipiert, organisiert und durchgeführt, war<br />

die erste Ausgabe <strong>de</strong>s Probestudiums ein so<br />

großer Erfolg, dass I<strong>de</strong>e und Konzept in <strong>de</strong>n<br />

folgen<strong>de</strong>n Jahren übernommen und weitergeführt<br />

wur<strong>de</strong>n. Die zentralen Bausteine, die<br />

2-stündigen Vorlesungen am Vormittag kombiniert<br />

mit Übungen in Kleingruppen an drei<br />

Nachmittagen, aufgelockert durch zwei Vorträge<br />

aus <strong>de</strong>r beruflichen Praxis und ergänzt<br />

durch eine Exkursion und eine Abschlussfeier<br />

mit Urkun<strong>de</strong>nverleihung, haben sich<br />

bewährt und blieben bis heute erhalten. Die<br />

Zuhörer wer<strong>de</strong>n so mit <strong>de</strong>m Ablauf typischer<br />

Lehrveranstaltungen <strong>de</strong>s Mathematikstudiums<br />

vertraut und lernen ein spannen<strong>de</strong>s<br />

Gebiet <strong>de</strong>r Mathematik näher kennen, beim<br />

letztjährigen Probestudium war es die Theorie<br />

<strong>de</strong>r komplexen Funktionen. Ergänzend<br />

wur<strong>de</strong>n 2009 <strong>de</strong>n Teilnehmern in Kurzvorträgen<br />

auch an<strong>de</strong>re Teilgebiete <strong>de</strong>r Mathematik<br />

vorgestellt, mit welchen sich unser <strong>Institut</strong><br />

beschäftigt.


21<br />

Das Probestudium ist keine Ein-Mann-Veranstaltung;<br />

<strong>de</strong>r organisatorische Aufwand<br />

ist beachtlich. Es müssen ja nicht nur Vorlesung<br />

und Übung konzipiert und gehalten<br />

wer<strong>de</strong>n, son<strong>de</strong>rn z.B. auch Gastvorträge organisiert,<br />

Flyer und Urkun<strong>de</strong>n gedruckt, Anmeldungen<br />

koordiniert, Mensakarten beschafft,<br />

Speisen und Getränke bestellt, Räume reserviert<br />

(keine allzu leichte Übung!), Studieren<strong>de</strong><br />

als Hilfskräfte gewonnen und eingewiesen<br />

wer<strong>de</strong>n und und und … Deswegen<br />

sei an dieser Stelle ganz herzlich all <strong>de</strong>njenigen<br />

gedankt, die an <strong>de</strong>n vergangenen<br />

10 Jahren Probestudium mitgewirkt<br />

haben, <strong>de</strong>n Sekretärinnen,<br />

Assistentinnen und<br />

Assistenten, Dozentinnen und<br />

Dozenten und ganz beson<strong>de</strong>rs<br />

auch allen Stu<strong>de</strong>ntinnen und<br />

Stu<strong>de</strong>nten, die als Tutorinnen<br />

und Tutoren Übungsgruppen<br />

betreut haben und die durch<br />

ihren Einsatz wesentlich zum<br />

Erfolg <strong>de</strong>s Probestudiums beigetragen<br />

haben.<br />

Das Probestudium ist – zusammen mit <strong>de</strong>m<br />

ebenfalls um die Jahrtausendwen<strong>de</strong> ins Leben<br />

gerufenen „Tag <strong>de</strong>r Mathematik“ und <strong>de</strong>r<br />

Aktion „Mathematik am Samstag“ – ein wichtiges<br />

Aushängeschild <strong>de</strong>s <strong>Institut</strong>s. Längst<br />

kommen die Teilnehmer<br />

nicht mehr nur aus <strong>de</strong>n<br />

Schulen, in <strong>de</strong>nen unser<br />

Flyer verteilt wird. Im letzten<br />

Jahr hatten sich sogar drei<br />

Schülerinnen aus Tschechien<br />

zum Probestudium<br />

angemel<strong>de</strong>t, sehr bedauernd,<br />

dass in ihrer Heimat<br />

nichts <strong>de</strong>rartiges angeboten<br />

wur<strong>de</strong>, und überlegen<br />

nun, in München an <strong>de</strong>r<br />

LMU Mathematik zu studieren.<br />

Das Team <strong>de</strong>s Probestudiums 2009<br />

So geht ein letzter und größter Dank an die Teilnehmerinnen<br />

und Teilnehmer <strong>de</strong>r vergangenen<br />

10 Jahre! Ohne <strong>de</strong>ren Interesse, Engagement<br />

und Bereitschaft, eine Woche Ferien für das Probestudium<br />

zu „opfern“, hätte es diese Veranstaltung<br />

keine 10 Jahre gegeben. Wir dürfen auf<br />

weitere 10 erfolgreiche Jahre hoffen!<br />

Daniel Rost


22<br />

Rätselecke<br />

Qua<strong>de</strong>r<br />

Schnei<strong>de</strong> ein Papierrechteck 3x4 nur so weit an, dass die<br />

durch Anschneidung entstan<strong>de</strong>nen Teile immer noch<br />

zusammenhalten und man mit <strong>de</strong>m Angeschnittenen<br />

Rechteck einen Qua<strong>de</strong>r 1x1x1 in zwei Schichten bekleben<br />

kann.<br />

Vielecke<br />

Auf einem Kreisumfang befin<strong>de</strong>n sich 12 blaue Punkte und ein roter<br />

Punkt. Gibt es mehr konvexe Vielecke mit nur blauen Ecken o<strong>de</strong>r mehr<br />

mit blauen und roten Ecken<br />

Gleise<br />

Auf <strong>de</strong>n Straßenbahngleisen liegt ein Quadrat, <strong>de</strong>ssen Seite gleich <strong>de</strong>m<br />

Abstand zwischen <strong>de</strong>n Gleisen ist. Wie groß ist <strong>de</strong>r Winkel zwischen<br />

zwei Gera<strong>de</strong>n, die überkreuzt die Schnittpunkte <strong>de</strong>r Quadratseiten mit <strong>de</strong>n<br />

Gleisen verbin<strong>de</strong>n<br />

<br />

Zauberschlacht<br />

Auf einem Feld befin<strong>de</strong>n sich 50 Zauberer. Mit nur einer Zauberstabbewegung verzaubert<br />

je<strong>de</strong>r <strong>de</strong>r Zauberer <strong>de</strong>n ihm am nächsten stehen<strong>de</strong>n Zauberer (d.h. einen <strong>de</strong>r am nächsten<br />

stehen<strong>de</strong>n Zauberer, falls sich mehrere Zauberer in gleicher Entfernung von ihm befin<strong>de</strong>n),<br />

wobei alle Zauberstabbewegungen gleichzeitig erfolgen. Wie viele Zauberer min<strong>de</strong>stens<br />

wer<strong>de</strong>n verzaubert


23<br />

Lösungen zu <strong>de</strong>n Rätseln von Ausgabe 20<br />

Würfel färben<br />

Je<strong>de</strong> Fläche eines Würfels ist in vier gleiche Quadrate geteilt. Wie viele unterschiedlich aussehen<strong>de</strong><br />

Würfel kann man erhalten, wenn man diese Quadrate in <strong>de</strong>n Farben Blau, Gelb o<strong>de</strong>r Rot so<br />

einfärbt, dass Quadrate mit einer gemeinsamen Seite verschie<strong>de</strong>n gefärbt sind Wie viele Quadrate<br />

von je<strong>de</strong>r Farbe gibt es dann<br />

Je<strong>de</strong>s <strong>de</strong>r 24 Teilquadrate grenzt an genau eine <strong>de</strong>r acht Ecken. Da nach unserer<br />

Vorschrift die drei Teilquadrate an je<strong>de</strong>r Ecke unterschiedlich gefärbt sein<br />

müssen, gibt es jeweils genau acht Teilquadrate in <strong>de</strong>n Farben Blau, Gelb und<br />

Rot.<br />

Es gibt genau sechs Möglichkeiten <strong>de</strong>r Einfärbung <strong>de</strong>r Seitenflächen: Wir<br />

bezeichnen die zweifarbigen mit I = blau/gelb, II = gelb/rot und III = rot/blau<br />

sowie die dreifarbigen mit (i), (ii) bzw. (iii), wenn die Farbe Rot, Blau bzw. Gelb<br />

doppelt vorkommt.<br />

Da jeweils acht blaue, gelbe und rote Teilquadrate vorkommen, sind nur folgen<strong>de</strong><br />

Kombinationen von Seitenflächen möglich:<br />

a) 2 ∙ I + 2 ∙ II + 2 ∙ III<br />

b) 2 ∙ I + 1 ∙ II + 1 ∙ III + 2 ∙ (i)<br />

2 ∙ II + 1 ∙ III + 1 ∙ I + 2 ∙ (ii)<br />

2 ∙ III + 1 ∙ I + 1 ∙ II + 2 ∙ (iii)<br />

c) 1 ∙ I + 1 ∙ II + 1 ∙ III + 1 ∙ (i) + 1 ∙ (ii) + 1 ∙ (iii)<br />

d) 1 ∙ I + 1 ∙ II + 2 ∙ (i) + 2 ∙ (ii)<br />

1 ∙ II + 1 ∙ III + 2 ∙ (ii) + 2 ∙ (iii)<br />

1 ∙ III + 1 ∙ I + 2 ∙ (iii) + 2 ∙ (i)<br />

e) 2 ∙ I + 4 ∙ (i)<br />

2 ∙ II + 4 ∙ (ii)<br />

2 ∙ III + 4 ∙ (iii)<br />

f) 1 ∙ I + 3 ∙ (i) + 1 ∙ (ii) + 1 ∙ (iii)<br />

1 ∙ II + 3 ∙ (ii) + 1 ∙ (iii) + 1 ∙ (i)<br />

1 ∙ III + 3 ∙ (iii) + 1 ∙ (i) + 1 ∙ (ii)<br />

g) 2 ∙ (i) + 2 ∙ (ii) + 2 ∙ (iii)<br />

Beim Beweis, welche Kombinationen unter Beachtung <strong>de</strong>r Vorschrift realisierbar sind und in<br />

welchen Konstellationen, helfen die folgen<strong>de</strong>n einfach zu beweisen<strong>de</strong>n Tatsachen: (1) Zwei<br />

gleiche zweifarbige Flächen können nicht benachbart sein. (2) Eine Kombination je einer Fläche<br />

vom Typ I, II und III hat eine gemeinsame Ecke.<br />

Insbeson<strong>de</strong>re unter Verwendung dieser Tatsachen folgt, dass es zwei Würfel vom Typ a), keinen<br />

vom Typ b) und f), vier Würfel vom Typ c), je sechs Würfel vom Typ d) und e) und sogar 21<br />

Würfel vom Typ g) gibt. Insgesamt erlaubt die Vorschrift also 39 verschie<strong>de</strong>n aussehen<strong>de</strong><br />

Würfel. Eine ausführliche Begründung ist nachzulesen auf <strong>de</strong>r Homepage <strong>de</strong>s För<strong>de</strong>rvereins<br />

(www.<strong>mathe</strong>matik.uni-muenchen.<strong>de</strong>/~fmwus).


24<br />

Ein Floß auf einem Kanal<br />

Ein Kanal <strong>de</strong>r Breite h hat an einer Stelle einen rechtwinkligen Knick. Welche maximale Fläche darf ein rechteckiges<br />

Floß besitzen, damit es diesen Kanal befahren kann<br />

Angenommen, das Floß ist ein Quadrat, dann kann seine Fläche<br />

höchstens h 2 betragen; das Floß passiert <strong>de</strong>n Kanal mithilfe von<br />

zwei Parallelverschiebungen.<br />

An<strong>de</strong>renfalls ist seine Länge größer als die Breite. In diesem Fall<br />

dreht sich das Floß um 90°, und in einem bestimmten Augenblick<br />

bil<strong>de</strong>n seine längeren Seiten mit <strong>de</strong>m Ufer <strong>de</strong>s Kanals<br />

einen 45°-Winkel. Es ist damit in ein großes gleichschenkligrechtwinkliges<br />

Dreieck mit <strong>de</strong>n Schenkellängen 2h und Hypotenuse<br />

2√2∙h einbeschrieben (siehe Abbildung).<br />

h<br />

x<br />

x<br />

h<br />

Sei x die Breite <strong>de</strong>s Floßes. Dann ist seine Länge 2√2∙h – 2x und seine Fläche ist gleich<br />

x∙(2√2∙h – 2x) = h 2 – 2(h/√2 – x) 2 . Der letzte Ausdruck nimmt sein Maximum für x = h/√2<br />

an. Das Floß mit <strong>de</strong>r maximalen Fläche h 2 besitzt <strong>de</strong>mnach Breite h/√2 und Länge √2∙h. Dieses<br />

Floß kann auf folgen<strong>de</strong> Weise <strong>de</strong>n Kanal passieren: Man lasse das Floß an <strong>de</strong>r Innenseite <strong>de</strong>s<br />

Kanals gleiten, bis die Mitte seiner Länge die herausragen<strong>de</strong> Ecke <strong>de</strong>s Kanals erreicht hat, dann<br />

drehe man das Floß um 90°. Dies ist möglich, da <strong>de</strong>r Abstand zwischen <strong>de</strong>r Seitenmitte <strong>de</strong>s<br />

Floßes und Punkten <strong>de</strong>r gegenüberliegen<strong>de</strong>n Seite höchstens h beträgt.<br />

Schachturnier<br />

Bei einem Schachturnier hat je<strong>de</strong>r Teilnehmer genauso viele Partien mit <strong>de</strong>n weißen Figuren gewonnen, wie <strong>de</strong>r<br />

Rest <strong>de</strong>r Teilnehmer mit <strong>de</strong>n schwarzen Figuren gewonnen hat. Stimmt es, dass alle Teilnehmer dieselbe Anzahl<br />

von Siegen feiern konnten<br />

Betrachten wir einen Turnierteilnehmer. Die Gesamtzahl seiner Siege ist die Summe <strong>de</strong>r Anzahl<br />

<strong>de</strong>r Partien, die er mit <strong>de</strong>n weißen Figuren gewonnen hat und <strong>de</strong>r Anzahl <strong>de</strong>r Partien, die er<br />

mit <strong>de</strong>n schwarzen Figuren gewonnen hat. Damit ist die Gesamtzahl seiner Siege gleich <strong>de</strong>r<br />

Anzahl <strong>de</strong>r Partien, die mit <strong>de</strong>n schwarzen Figuren gewonnen wur<strong>de</strong>n; diese Anzahl ist aber<br />

für alle Turnierteilnehmer gleich.<br />

Flächen<br />

Welcher Flächeninhalt ist größer, <strong>de</strong>r <strong>de</strong>s grau gefärbten Ringes o<strong>de</strong>r <strong>de</strong>rjenige<br />

<strong>de</strong>s schraffierten Kreises<br />

Die Flächeninhalte sind gleich groß. Sei r <strong>de</strong>r Radius <strong>de</strong>s inneren<br />

weißen Kreises und gleichzeitig <strong>de</strong>r Radius <strong>de</strong>s schraffierten<br />

Kreises. Der Radius R <strong>de</strong>s großen Kreises ist dann gleich √2r und<br />

<strong>de</strong>r Flächeninhalt <strong>de</strong>s grauen Ringes ist gleich πR 2 – πr 2 = πr 2<br />

und damit gleich <strong>de</strong>r Fläche <strong>de</strong>s schraffierten Kreises.


Der Zufall – ein Helfer und kein Störenfried<br />

Warum die Wissenschaft stochastische<br />

Mo<strong>de</strong>lle braucht<br />

Götz Kersting<br />

Der Zufall hat in <strong>de</strong>n Wissenschaften weithin<br />

einen zweifelhaften Ruf. Für die Philosophie<br />

hat Hegel festgestellt: „Die philosophische<br />

Betrachtung hat keine an<strong>de</strong>re Absicht,<br />

als das Zufällige zu entfernen“ (Die Vernunft<br />

in <strong>de</strong>r Geschichte, 1822) – und ähnlich <strong>de</strong>nkt<br />

man auch in an<strong>de</strong>ren Wissenschaften. Die<br />

Auseinan<strong>de</strong>rsetzungen <strong>de</strong>r Physik mit <strong>de</strong>m<br />

Zufall sind verschlungen und bis heute von<br />

Kontroversen begleitet. Was die Biologie<br />

betrifft, so herrscht noch einiger Argwohn<br />

gegenüber <strong>de</strong>n mo<strong>de</strong>rnen Evolutionstheorien,<br />

die sich entschei<strong>de</strong>nd auf <strong>de</strong>n Zufall<br />

stützen. Und dass <strong>de</strong>rartige Theorien unvereinbar<br />

sind mit <strong>de</strong>r Vorstellung von einer<br />

göttlichen Schöpfung <strong>de</strong>r Welt, gilt unter<br />

manchen ihrer Gegner wie Befürworter als<br />

ausgemacht.<br />

Die Skepsis rührt zu einem gewichtigen Teil<br />

daher, dass <strong>de</strong>r Zufall kaum als ein eigenständiges<br />

Konzept wahrgenommen wird, son<strong>de</strong>rn<br />

nur als Gegensatz zum Geregelten, Geordneten,<br />

also als ein Störenfried. Des Zufalls „blin<strong>de</strong>s“<br />

Wirken scheint sich schlecht damit zu<br />

vertragen, dass sich das unserem Verständnis<br />

zugängliche Geschehen in <strong>de</strong>r Welt <strong>de</strong>terministisch<br />

vollzieht. So gilt als vorbildlich für die<br />

Wissenschaften, wie es Kepler verstand, das<br />

Zufällige in <strong>de</strong>n Planetenbewegungen, wie<br />

man das am Himmel beobachtet, auf einfache<br />

Gesetzmäßigkeiten zurückzuführen und<br />

damit zu eliminieren. Dem fühlt sich auch<br />

die mo<strong>de</strong>rne Chaostheorie verpflichtet, und<br />

man hat sich noch nicht überall damit abgefun<strong>de</strong>n,<br />

dass man manchmal – nicht nur in<br />

<strong>de</strong>r Quantentheorie – ohne <strong>de</strong>n Zufall nicht<br />

auskommt.<br />

„Der Zufall ist zu etwas in <strong>de</strong>r Lage, was man<br />

gewöhnlich nur <strong>de</strong>terministischen Handlungsvorschriften<br />

zutraut.“<br />

Aber wie begrün<strong>de</strong>t sind solche Vorbehalte<br />

Es erweist sich nämlich, dass <strong>de</strong>r Zufall oft gar<br />

nicht als Störenfried in Erscheinung tritt, son<strong>de</strong>rn<br />

vielmehr als Helfer wirkt, <strong>de</strong>m erstaunliche<br />

Dinge gelingen. Dem wollen wir in drei<br />

Beispielen nachgehen. Im ersten Beispiel zeigt<br />

sich, dass bestimmte Aufgabenstellungen<br />

besser gelöst wer<strong>de</strong>n können, wenn man <strong>de</strong>n<br />

Zufall zu Hilfe nimmt. Es geht um das Auflösen<br />

von Warteschlangen beim Hin- und Herschicken<br />

von Paketen in komplexen Netzwerken,<br />

konkret gesprochen von Datenpaketen<br />

im Computer. Wie sich herausstellt, vermei<strong>de</strong>t<br />

man Warteschlangen am zuverlässigsten,<br />

wenn man die Routen für die Pakete in geeigneter<br />

Weise zufällig wählt. Hier ist <strong>de</strong>r Zufall<br />

zu etwas in <strong>de</strong>r Lage, was man gewöhnlich<br />

nur <strong>de</strong>terministischen Handlungsvorschriften<br />

zutraut. In <strong>de</strong>r Informatik macht man sich<br />

diese Qualität <strong>de</strong>s Zufalls inzwischen häufig<br />

zunutze und entwirft randomisierte Algorithmen<br />

für verschie<strong>de</strong>nste Zwecke.<br />

„Kausales Denken geht ein Bündnis mit <strong>de</strong>m<br />

Zufall ein.“<br />

Im zweiten Beispiel geht es um ein Problem<br />

aus <strong>de</strong>r Statistik. Wie bei vielen an<strong>de</strong>ren<br />

statistischen Problemen will man feststellen,<br />

ob sich in einem Datensatz ein ursächlicher<br />

Zusammenhang auf<strong>de</strong>cken lässt. Häufig<br />

ist das aber nicht direkt zu klären: Einerseits<br />

fin<strong>de</strong>t sich in <strong>de</strong>n Daten Variabilität, an<strong>de</strong>rerseits<br />

bereitet es Schwierigkeiten, mögliche<br />

Wirkungszusammenhänge explizit zu benennen.<br />

Deswegen dreht man <strong>de</strong>n Spieß um und<br />

befasst sich mit <strong>de</strong>r Frage, ob die Daten rein<br />

25


erkläh<br />

ein<br />

t ausrweisthetischen,<br />

vom Zufall gesteuerten Mo<strong>de</strong>llen, Stellen wir uns vor, dass in einem Netzwerk<br />

26<br />

Insgesamt ist festzustellen, dass die Wissenschaft <strong>de</strong>n<br />

Zufall als eigenständiges Konzept verstehen und entwickeln<br />

muss. Diese Aufgabe ist Gegenstand <strong>de</strong>r Stochastik<br />

(»Kunst <strong>de</strong>s Mutmaßens«), <strong>de</strong>r <strong>mathe</strong>matischen<br />

Lehre vom Zufall. In ihr geht es weniger darum, ob es<br />

zufällig <strong>de</strong>n entstan<strong>de</strong>n Zufall in <strong>de</strong>r sein Welt könnten. wirklich Das ist gibt, einfacher:<br />

Aufgabe, Man konfrontiert <strong>de</strong>n Begriff die Daten <strong>de</strong>s Zufalls mit hypo-<br />

herauszuarbeiten.<br />

Besser mithilfe <strong>de</strong>s Zufalls<br />

vielmehr Auflösen ist von es Warteschlangen:<br />

ihre<br />

ören-auch wenn Auflösen man sie von faktisch Warteschlangen: gar nicht ernsthaft<br />

in Besser Betracht mithilfe zieht. Kann <strong>de</strong>s Zufalls man glaubhaft schie<strong>de</strong>nen Knotenpunkten treffen sie auf-<br />

Pakete hin- und hergeschickt wer<strong>de</strong>n. An ver-<br />

Helfer<br />

ol len machen, Stellen dass sich wir die uns Daten vor, so dass nicht in erklären<br />

hin- lassen, und so hergeschickt gilt dies als Beleg, wer<strong>de</strong>n. dass eben An verschie<strong>de</strong>nen an<strong>de</strong>ren weitergeleitet Kno-<br />

wer<strong>de</strong>n (zum Beispiel<br />

einem einan<strong>de</strong>r, Netzwerk sie Pakete müssen dann eines nach <strong>de</strong>m<br />

ispiel<br />

doch ein Wirkungszusammenhang besteht. nach <strong>de</strong>r Regel FIFO: „first in first out“). Wie<br />

besser tenpunkten treffen sie aufeinan<strong>de</strong>r, sie müssen dann<br />

Hier geht kausales Denken ein Bündnis mit kann man verhin<strong>de</strong>rn, dass dabei lange Warteschlangen<br />

Hilfe<br />

<strong>de</strong>m<br />

eines<br />

Zufall<br />

nach<br />

ein; die<br />

<strong>de</strong>m<br />

Statistik<br />

an<strong>de</strong>ren<br />

mit ihren<br />

weitergeleitet<br />

Zufallsbetrachtungen<br />

Beispiel nach wird zum <strong>de</strong>r methodischen Regel FIFO: Rüst-<br />

»first in Informationsfluss first out«). Wie zusammenbricht Offenbar<br />

wer<strong>de</strong>n<br />

entstehen;<br />

(zum<br />

Knoten, an <strong>de</strong>nen <strong>de</strong>r<br />

angen<br />

lexen zeug kann einer man empirischen verhin<strong>de</strong>rn, Wissenschaft, dass dabei <strong>de</strong>r es lange han<strong>de</strong>lt Warteschlangen<br />

sich um eine Fragestellung aus <strong>de</strong>r<br />

en im gar entstehen; nicht um <strong>de</strong>n Knoten, Zufall geht. an <strong>de</strong>nen Das hat <strong>de</strong>r auch Informationsfluss Welt <strong>de</strong>r Rechner zusammenbricht<br />

Tücken, <strong>de</strong>nnoch Offenbar ist die Entwicklung han<strong>de</strong>lt es sich um eine Fra-<br />

und <strong>de</strong>s Internets.<br />

War-seinouten <strong>de</strong>r gestellung Statistik eine aus mo<strong>de</strong>rne Welt Erfolgsgeschichte. <strong>de</strong>r Rechner und Schauen <strong>de</strong>s Internets. wir auf ein Beispiel, ein Netzwerk<br />

. Hier Schauen wir auf ein Beispiel, ein Netzwerk von <strong>de</strong>r Gestalt von eines <strong>de</strong>r Würfels, wie in Abb. 1.<br />

öhnn<br />

zu-<br />

Die Ecken <strong>de</strong>s Würfels sind die<br />

Im letzten Beispiel schließlich <strong>de</strong>uten wir an,<br />

Gestalt eines Würfels, wie in 1.<br />

wie man in <strong>de</strong>r Finanz<strong>mathe</strong>matik <strong>de</strong>m Börsengeschehen<br />

und seinen Risiken mit Zufalls-<br />

111<br />

011<br />

uali-mo<strong>de</strong>llen twirft um bart, Fragestellungen, wenn sie die durch erst einmal eine Kante gar nichts ver-<br />

101<br />

Kno tenpunkte. auf <strong>de</strong>n Grund Sie geht. heißen Dabei benach-<br />

geht es<br />

001<br />

ecke. mit bun<strong>de</strong>n Zufall zu tun sind. haben, Die auch Kanten entsprechen sind die<br />

Zufallsmo<strong>de</strong>lle Verbindungen, <strong>de</strong>finitiv nicht entlang <strong>de</strong>r Wirklichkeit. <strong>de</strong>rer die<br />

Dennoch Pakete ist in es hier bei<strong>de</strong> so, Richtungen dass diese „falschen“ laufen<br />

010<br />

Mo<strong>de</strong>lle (sogenannte „Martingalmaße“) das<br />

110<br />

können, pro Takteinheit aber immer<br />

Verständnis nur jeweils entschei<strong>de</strong>nd eines. för<strong>de</strong>rn. Zwischen Das klingt zwei 000<br />

paradox und ist nicht ganz leicht zu vermitteln.<br />

An diesen Mo<strong>de</strong>llen kommt man aber<br />

100<br />

s <strong>de</strong>r nicht benachbarten Ecken gibt es<br />

roblensatz<br />

<strong>de</strong>n aus Finanzmärkten höchstens in drei Schranken Kanten. halten. Stellen Spe-<br />

wir uns weiter vor, 1 Einfaches Mo-<br />

Abb. 1: Einfaches Mo<strong>de</strong>ll eines Netzwerkes mit acht<br />

nicht<br />

ver<br />

mehr<br />

schie<strong>de</strong>ne<br />

vorbei, will<br />

Verbindungswege,<br />

man die Risiken auf<br />

die direkten bestehen<br />

Knoten und dreistelligen digitalen Adressen.<br />

. Häu-zialistefin<strong>de</strong>t immer an<strong>de</strong>ren wichtigere Ecke Rolle gesen<strong>de</strong>t zu. wird. Dabei te. entstehen Sie heißen dann benachbart,<br />

dass aus von <strong>de</strong>r je<strong>de</strong>r Mathematik <strong>de</strong>r acht wächst Ecken hier ein eine Paket Die zu Ecken irgen<strong>de</strong>iner <strong>de</strong>s Würfels sind<br />

<strong>de</strong>ll<br />

die<br />

eines<br />

Knotenpunk-<br />

Netzwerkes<br />

wenn<br />

mit<br />

sie<br />

acht<br />

durch<br />

Knoten und dreistelligen<br />

digitalen<br />

itet es Schlangen bis zur Länge acht. Dies ist eine kleines, Kante verbun<strong>de</strong>n noch sind. Die Kanten sind<br />

hänge Insgesamt nicht ist beson<strong>de</strong>rs festzustellen, interessantes dass die Wissenschaft Netzwerk. die Verbindungen, entlang <strong>de</strong>rer die Pakete<br />

Adressen.<br />

Spieß <strong>de</strong>n Zufall Spannen<strong>de</strong>r als eigenständiges sind Konzept große verstehen Netzwerke. in Wir bei<strong>de</strong> betrachten Richtungen laufen können, pro Takteinheit<br />

und entwickeln muss. Diese Aufgabe ist Gegenstand<br />

<strong>de</strong>r Stochastik („Kunst <strong>de</strong>s Mutmaßens“), schen zwei nicht benachbarten Ecken gibt es<br />

rein nun Netzwerke, bei <strong>de</strong>m je<strong>de</strong>r Knoten eine<br />

aber<br />

Adresse<br />

immer nur jeweils eines. Zwi-<br />

cher: aus n Zahlen x<br />

<strong>de</strong>r <strong>mathe</strong>matischen 1<br />

x<br />

Lehre 2<br />

…x<br />

vom n<br />

besitzt, wobei je<strong>de</strong> dieser n Zahlen<br />

es weniger entwe<strong>de</strong>r darum, <strong>de</strong>n ob Wert es <strong>de</strong>n 0 Zufall o<strong>de</strong>r in 1 <strong>de</strong>r annimmt.<br />

Zufall. In ihr verschie<strong>de</strong>ne Verbindungswege, die direkten<br />

, vom geht bestehen<br />

Damit<br />

aus höchstens<br />

hat<br />

drei Kanten. Stellen<br />

n = wir 3 entspricht, uns weiter vor, dass von je<strong>de</strong>r <strong>de</strong>r<br />

e fak-Welman <strong>de</strong>n wie Begriff man <strong>de</strong>s Zufalls Abbildung herauszuarbeiten. 1 entnimmt, einem acht drei-dimensi-<br />

Ecken ein Paket zu irgen<strong>de</strong>iner man wirklich 2 n verschie<strong>de</strong>ne gibt, vielmehr ist Adressen. ihre Aufgabe, Der Fall<br />

an<strong>de</strong>ren<br />

onalen Würfel. Der Abbildung entnimmt man auch,<br />

dass zwei Ecken genau dann benachbart sind, wenn


öchstens <strong>de</strong>r Länge n verbun<strong>de</strong>n,<br />

unterschei<strong>de</strong>n sich an höchstens<br />

ensionale Würfel ist in <strong>de</strong>r Abbil-<br />

0010<br />

0110<br />

1010<br />

1110<br />

27<br />

Algorithmus<br />

Ecke gesen<strong>de</strong>t<br />

provoziert<br />

wird.<br />

Staus<br />

Dabei entstehen<br />

dann Schlangen bis zur<br />

0100<br />

un also die folgen<strong>de</strong> Aufgabe: Von<br />

1100<br />

n soll<br />

Länge<br />

ein Paket<br />

acht.<br />

an<br />

Dies<br />

eine<br />

ist<br />

an<strong>de</strong>re<br />

ein kleines,<br />

A<strong>de</strong>n,<br />

unter <strong>de</strong>n oben genannten Be-<br />

1000<br />

noch nicht beson<strong>de</strong>rs interessantes<br />

0000<br />

immt <strong>de</strong>r<br />

Netzwerk.<br />

Übersichtlichkeit halber<br />

0111<br />

1111<br />

die 2 n Zieladressen alle verschie<strong>de</strong>n<br />

Gibt<br />

Spannen<strong>de</strong>r<br />

es günstige Regeln,<br />

sind große<br />

bei <strong>de</strong>nen<br />

Netzwerke. Wir<br />

0011<br />

betrachten nun Netzwerke, bei <strong>de</strong>nen je<strong>de</strong>r<br />

1011<br />

schlangen entstehen Realistisch ist<br />

, dass<br />

Knoten<br />

die Regel,<br />

eine<br />

nach<br />

Adresse<br />

<strong>de</strong>r je<strong>de</strong>s<br />

aus n<br />

einart<br />

zum Ziel geschickt wird, unab-<br />

Zahlen x 1<br />

x 2<br />

...x n<br />

besitzt, wobei je<strong>de</strong> dieser n Zahlen 2 entwe<strong>de</strong>r<br />

Wert 0 o<strong>de</strong>r 1 annimmt. Damit eines Netzwerkes hat<br />

0101<br />

Darstellung<br />

as mit <strong>de</strong>n an<strong>de</strong>ren Paketen gen<br />

also, bildlich verschie<strong>de</strong>ne gesprochen, Adressen. unser Der Fall n = 3<br />

man 2 n mit 16 Knoten als<br />

1101<br />

vier-dimensionaler 0001<br />

euklappen-Algorithmen.<br />

entspricht, wie man Abbildung 1 entnimmt, Würfel.<br />

1001<br />

ressante einem <strong>mathe</strong>matische drei-dimensionalen Resultate. Würfel. Der Abbildung<br />

<strong>de</strong>terministische entnimmt man Scheuklap-<br />

auch, dass zwei Ecken<br />

Abb. 2: Darstellung eines Netzwerkes mit 16 Knoten<br />

einmal als vier-dimensionaler Würfel.<br />

avon genau gibt es dann eine benachbart Vielzahl, nahe sind, lielgen<strong>de</strong><br />

Adressen Verfahren: an Soll genau ein einer Paket Stelle von unterscheisen<br />

braucht man Regeln, min<strong>de</strong>stens bei <strong>de</strong>nen 2keine n/2 /n Schritte, langen Warteschlan-<br />

bis schließ-<br />

wenn sei sich gesagt: ihre Bei ungünstiger Konstellation <strong>de</strong>r Zieladres-<br />

z <strong>de</strong>n. So wollen wir es auch allgemein im gen entstehen Realistisch ist die Randbedingung,<br />

diese dass Zahl die sehr Regel, groß, nach <strong>de</strong>r im je<strong>de</strong>s Fall n einzel-<br />

= 100<br />

n<br />

geschickt wer<strong>de</strong>n, so suche man lich alle Pakete zugestellt sind. In sehr großen Netzwerken<br />

...x n<br />

Fall n > 3 halten: Die Ecken x 1<br />

x 2<br />

und wird<br />

y 1<br />

y 2<br />

...y n<br />

seien benachbart, sind also größer durch als 10ne 13 . Paket Dabei vom hat Start man zum zwischen Ziel geschickt zwei beliebig wird,<br />

eine Kante verbun<strong>de</strong>n, wenn sich ausgewählten bei<strong>de</strong> unabhängig Adressen einen davon Verbindungsweg, ist, was mit <strong>de</strong>n an<strong>de</strong>ren <strong>de</strong>ssen<br />

Adressen an genau einer Stelle unterschei<strong>de</strong>n.<br />

Es stellt sich gesprochen, weiter heraus: unser Augenmerk Dieses entmutigen<strong>de</strong> auf Scheu-<br />

Länge höchstens Paketen 100 geschieht. ist! Wir richten also, bildlich<br />

Resultat gilt für klappen-Algorithmen.<br />

je<strong>de</strong>n <strong>de</strong>terministischen Scheuklappen-<br />

Dieses Netzwerk heißt n-dimensionaler Algorithmus. Immer lassen sich Zieladressen angeben,<br />

Würfel. Er hat 2 n Ecken und n2 n–1 so Kanten, dass zumin<strong>de</strong>st Dazu gibt <strong>de</strong>r es Größenordnung interessante <strong>mathe</strong>matische<br />

nach die beschriebene<br />

Weg ungünstige Resultate. Betrachten Situation wir besteht. erst einmal <strong>de</strong>termi-<br />

und je zwei Ecken sind durch einen<br />

höchstens <strong>de</strong>r Länge n verbun<strong>de</strong>n, <strong>de</strong>nn ihre nistische Scheuklappen-Algorithmen. Davon<br />

Adressen unterschei<strong>de</strong>n sich an höchstens Mit zufällig n gibt gewählten es eine Vielzahl, Zwischenstopps nahe liegend ist etwa<br />

Stellen. Der 4-dimensionale Würfel ist schneller in <strong>de</strong>r das ans Ziel folgen<strong>de</strong> Verfahren: Soll ein Paket von<br />

Abbildung 2 dargestellt.<br />

Der Zufall x 1 macht x 2<br />

...x n<br />

nach es möglich, z 1<br />

z 2<br />

...z n diesen geschickt schwer wer<strong>de</strong>n, zu fassen<strong>de</strong>n<br />

ungünstigen suche man Konstellationen erst die kleinste Stelle auszuweichen.<br />

i, an <strong>de</strong>r sich<br />

so<br />

Deterministischer Algorithmus provoziert Zum Beispiel x i auf und die z i<br />

unterschei<strong>de</strong>n, folgen<strong>de</strong> Weise: und Zu schicke je<strong>de</strong>r das Adresse Paket<br />

Staus<br />

wähle man, zunächst zusätzlich an <strong>de</strong>n zu <strong>de</strong>r benachbarten Zieladresse, Knoten, noch <strong>de</strong>ssen eine<br />

Wir betrachten nun also die folgen<strong>de</strong> Zwischenadresse. Aufgabe:<br />

Von je<strong>de</strong>r <strong>de</strong>r 2 n Adressen soll zufällig ein Paket gewählt, Startadresse unabhängig unterschei<strong>de</strong>t. Er hat die Adresse<br />

Adresse Diese sich Zwischenadressen an genau dieser Stelle wer<strong>de</strong>n von rein <strong>de</strong>r<br />

an eine an<strong>de</strong>re Adresse geschickt wer<strong>de</strong>n, x 1<br />

...x i–1<br />

z i<br />

x i+1<br />

...x n<br />

= z ...z von z Knoten x ...x zu Knoten.<br />

Dann verschicke man die Pakete 1 i–1durch i i+1 sAdA, n<br />

. Dann jedoch ersetze<br />

an man die die Zwischenadressen! nächste unterschiedliche Sind Stelle diese und er-<br />

unter <strong>de</strong>n oben genannten Bedingungen. erst einmal<br />

Man nimmt <strong>de</strong>r Übersichtlichkeit halber reicht, gerne so verschicke so weiter, man bis zerneut 1<br />

z 2<br />

...z n<br />

erreicht die Pakete ist. Verfährt durch sAdA, man<br />

an, dass auch die 2 n Zieladressen nun alle an verschie<strong>de</strong>n<br />

sind. Die Frage ist: Gibt es sierte günstige Scheuklappen-Algorithmus so hat man einen wohl<strong>de</strong>finierten braucht, mit Scheuklap- kleiner<br />

die Zieladressen. bei Warteschlangen Es zeigt noch sich: nach Dieser <strong>de</strong>r Regel randomi-<br />

FIFO,<br />

Ausnahmewahrscheinlichkeit, nur höchstens 6.2 × n<br />

Schritte, bis alle Pakete zugestellt sind. Genauer lässt


28<br />

pen-Algorithmus, nennen wir ihn sukzessives<br />

Ausrichten <strong>de</strong>r Adressen (sAdA).<br />

Nun stellt sich heraus: Wenn man die Zieladressen<br />

unglücklich wählt, entstehen beim<br />

sAdA lange Warteschlangen. Wie das im Einzelnen<br />

aussieht, damit wollen wir uns hier<br />

nicht auseinan<strong>de</strong>rsetzen, nur so viel sei gesagt:<br />

Bei ungünstiger Konstellation <strong>de</strong>r Zieladressen<br />

braucht man min<strong>de</strong>stens 2 n/2 /n Schritte,<br />

bis schließlich alle Pakete zugestellt sind. In<br />

sehr großen Netzwerken wird diese Zahl sehr<br />

groß, im Fall n = 100 größer als 10 13 . Dabei<br />

hat man zwischen zwei beliebig ausgewählten<br />

Adressen einen Verbindungsweg, <strong>de</strong>ssen<br />

Länge höchstens 100 ist!<br />

Es stellt sich weiter heraus: Dieses entmutigen<strong>de</strong><br />

Resultat gilt für je<strong>de</strong>n <strong>de</strong>terministischen<br />

Scheuklappen-Algorithmus. Immer lassen<br />

sich Zieladressen angeben, so dass zumin<strong>de</strong>st<br />

<strong>de</strong>r Größenordnung nach die beschriebene<br />

ungünstige Situation besteht.<br />

Mit zufällig gewählten Zwischenstopps<br />

schneller ans Ziel<br />

Der Zufall macht es möglich, diesen schwer<br />

zu fassen<strong>de</strong>n ungünstigen Konstellationen<br />

auszuweichen. Zum Beispiel auf die folgen<strong>de</strong><br />

Weise: Zu je<strong>de</strong>r Adresse wähle man, zusätzlich<br />

zu <strong>de</strong>r Zieladresse, noch eine Zwischenadresse.<br />

Diese Zwischenadressen wer<strong>de</strong>n rein zufällig<br />

gewählt, unabhängig von Knoten zu Knoten.<br />

Dann verschicke man die Pakete durch sAdA,<br />

jedoch erst einmal an die Zwischenadressen!<br />

Sind diese erreicht, so verschicke man erneut<br />

die Pakete durch sAdA, nun an die Zieladressen.<br />

Es zeigt sich: Dieser randomisierte<br />

Scheuklappen-Algorithmus braucht, mit kleiner<br />

Ausnahmewahrscheinlichkeit, nur höchstens<br />

6.2 × n Schritte, bis alle Pakete zugestellt<br />

sind. Genauer lässt sich die Ausnahmewahrscheinlichkeit<br />

abschätzen als<br />

Ws(mehr als 6.2 × n Schritte) < 2 –(n–1) .<br />

Im Fall n = 100 benötigt also <strong>de</strong>r randomisierte<br />

Algorithmus höchstens 620 Schritte,<br />

abgesehen vom Ausnahmefall, <strong>de</strong>r eine Wahrscheinlichkeit<br />

von weniger als 10 –29 besitzt.<br />

Dieses Resultat be<strong>de</strong>utet: Auch wenn es<br />

schwer zu sagen ist, welche Zieladressen zu<br />

langen Wartezeiten führen, so ist ihre relative<br />

Häufigkeit gering. Man kann ihnen daher<br />

durch rein zufällige Wahl <strong>de</strong>r Zieladressen<br />

ausweichen.<br />

Kann das <strong>de</strong>nn Zufall sein<br />

Ein Beispiel aus <strong>de</strong>r Statistik<br />

Die Statistik hat das Anliegen, kausale Zusammenhänge,<br />

<strong>de</strong>terministische Einflüsse, die<br />

in irgendwelchen Daten zum Ausdruck zu<br />

kommen scheinen, glaubhaft zu machen. Dabei<br />

ist es nicht ihr Weg, solche kausalen Beziehungen<br />

aufzuzeigen, sie möchte umgekehrt aufzeigen,<br />

dass <strong>de</strong>r Zufall als Erklärungsmuster für<br />

die Daten nicht taugt.<br />

Wir wollen diese Vorgehensweise <strong>de</strong>r Statistik<br />

an einem Beispiel <strong>de</strong>monstrieren: Eine Botschaft<br />

ein und <strong>de</strong>sselben Inhalts (es ging um<br />

<strong>de</strong>n Vergleich <strong>de</strong>s Erfolgs zweier Therapiemetho<strong>de</strong>n<br />

(T1 und T2)) wur<strong>de</strong> in zwei unterschiedliche<br />

Darstellungsformen verpackt. In<br />

Form A wur<strong>de</strong> herausgestellt, wie groß jeweils<br />

<strong>de</strong>r Prozentsatz <strong>de</strong>r Patienten ist, bei <strong>de</strong>nen<br />

Behandlung T1 erfolglos bzw. Behandlung T2<br />

erfolgreich war, in Form B wur<strong>de</strong> <strong>de</strong>r Akzent<br />

gera<strong>de</strong> umgekehrt gesetzt.<br />

Von insgesamt 167 Ärzten, die an einer Sommerschule<br />

teilnahmen, wur<strong>de</strong>n rein zufällig 80<br />

ausgewählt, <strong>de</strong>nen die Botschaft in <strong>de</strong>r Form


29<br />

A vermittelt wur<strong>de</strong>, die restlichen 87 bekamen<br />

die Botschaft in <strong>de</strong>r Form B mitgeteilt. Je<strong>de</strong>r<br />

<strong>de</strong>r Ärzte hatte sich daraufhin für die Bevorzugung<br />

einer <strong>de</strong>r bei<strong>de</strong>n Therapiemetho<strong>de</strong>n zu<br />

entschei<strong>de</strong>n. Das Ergebnis war:<br />

für Metho<strong>de</strong> T1: für Metho<strong>de</strong> T2: Summe:<br />

A: 40 40 80<br />

B: 73 14 87<br />

Summe: 113 54 167<br />

Die Daten zeigen: In <strong>de</strong>r A-Gruppe gibt es<br />

im Verhältnis weniger Befürworter <strong>de</strong>r Therapiemetho<strong>de</strong><br />

T1 als in <strong>de</strong>r B-Gruppe (nämlich<br />

40:40 gegen 73:14). Daher könnte <strong>de</strong>r Verdacht<br />

entstehen, dass die Art, in <strong>de</strong>r die Botschaft<br />

mitgeteilt wur<strong>de</strong>, ten<strong>de</strong>nziös war und<br />

die Ärzte sich dadurch in ihrem Urteil haben<br />

beeinflussen lassen. Ein Skeptiker könnte einwen<strong>de</strong>n:<br />

„Ein <strong>de</strong>rartiges Ergebnis kann auch<br />

ohne Beeinflussung zustan<strong>de</strong> kommen, wenn<br />

<strong>de</strong>r Zufall es will.“<br />

Haben die Ärzte sich täuschen lassen<br />

O<strong>de</strong>r war es ein Zufall<br />

Ob die Botschaft wirklich ten<strong>de</strong>nziös war, ist<br />

nicht so einfach zu klären. Die Behauptung<br />

<strong>de</strong>s Skeptikers lässt sich besser überprüfen<br />

und gegebenenfalls wi<strong>de</strong>rlegen. Dazu führen<br />

wir folgen<strong>de</strong> Hypothese über <strong>de</strong>n Zufall ein:<br />

Die Form <strong>de</strong>r Botschaft habe keinen Einfluss<br />

auf die Meinungsbildung <strong>de</strong>r 167 Ärzte<br />

gehabt; es wäre so, als ob die einen 80 die<br />

Botschaft auf rotem, die an<strong>de</strong>ren 87 eine<br />

wörtlich gleichlauten<strong>de</strong> Botschaft auf blauem<br />

Papier bekommen hätten. Wie wahrscheinlich<br />

ist es, dass eine so extreme Aufteilung in 113<br />

Befürworter von T1 und nur 54 Befürworter<br />

von T2 zufällig zustan<strong>de</strong> kommt<br />

Eine Veranschaulichung: Wenn aus einer<br />

Urne mit 80 roten und 87 blauen Kugeln<br />

rein zufällig 113 Kugeln gezogen wer<strong>de</strong>n,<br />

wie wahrscheinlich ist dann ein so extremes<br />

Ergebnis wie das, nur 40 rote Kugeln zu<br />

ziehen Wir wollen die Analyse hier nicht im<br />

Detail durchführen, auch wenn sie nicht sehr<br />

schwierig ist; (Stichworte sind Erwartungswert<br />

und Varianz <strong>de</strong>r hypergeometrischen Verteilung).<br />

Das Resultat ist, dass in <strong>de</strong>r Stichprobe<br />

im Mittel 54 rote Kugeln enthalten sind und<br />

dass eine Abweichung größer o<strong>de</strong>r gleich 14<br />

nur mit Wahrscheinlichkeit 6 × 10 –6 eintritt,<br />

6-mal in einer Million Fälle.<br />

Der Zufall schei<strong>de</strong>t damit nach menschlichem<br />

Ermessen als Erklärungsgrund für die<br />

in <strong>de</strong>n Daten sichtbare Ten<strong>de</strong>nz aus. Die Versuchspersonen<br />

haben sich, wie es aussieht,<br />

beeinflussen lassen. Wir sehen, <strong>de</strong>r Zufall<br />

dient in diesem Beispiel allein als Gedankenexperiment.<br />

Es hilft, ihn zurate zu ziehen,<br />

auch wenn in Wirklichkeit etwas ganz an<strong>de</strong>res<br />

geschehen ist. Von solch hypothetischer<br />

Bauart sind viele Überlegungen innerhalb <strong>de</strong>r<br />

Statistik.<br />

Der Zufall in <strong>de</strong>r Finanz<strong>mathe</strong>matik<br />

Das genuine Thema <strong>de</strong>r mo<strong>de</strong>rnen Finanz<strong>mathe</strong>matik<br />

sind Termingeschäfte und ihre<br />

Bewertung. Zufallsüberlegungen spielen dabei<br />

eine zentrale Rolle, jedoch in ganz an<strong>de</strong>rer<br />

Weise, als man zunächst meinen könnte.<br />

Worum es geht, wird schon im einfachsten<br />

Fall <strong>de</strong>utlich.<br />

Stellen wir uns vor, dass ein Terminkontrakt<br />

in einer zukünftigen Auszahlung besteht,<br />

entwe<strong>de</strong>r einer größeren Auszahlung t ↑ o<strong>de</strong>r<br />

einer kleineren t ↓ . Welche das sein wird,<br />

hängt von <strong>de</strong>r Entwicklung eines Aktienkurses<br />

ab, für <strong>de</strong>n wir auch zwei zukünftige<br />

Werte in Betracht ziehen, einen höheren a ↑


30<br />

und einen niedrigeren a ↓ . Was ist ein angemessener<br />

Preis P für einen solchen Kontrakt<br />

Wie steht er zum aktuellen Kurs a <strong>de</strong>r Aktie<br />

(a ↓ < a < a ↑ )<br />

Eine plausible Antwort lautet: Das kommt auf<br />

die Erwartungshaltung <strong>de</strong>r Akteure an. Ist<br />

w ↑ die Wahrscheinlichkeit, dass <strong>de</strong>r Aktienkurs<br />

auf a ↑ steigt – und damit w ↓ = 1 – w ↑<br />

diejenige, dass <strong>de</strong>r Kurs auf a ↓ sinkt, – so ist<br />

es natürlich, <strong>de</strong>n Preis als das mit <strong>de</strong>n Wahrscheinlichkeiten<br />

gewichtete Mittel von t ↑ und<br />

t ↓ anzusetzen, also als<br />

P(w ↑ , w ↓ ) = t ↑ w ↑ + t ↓ w ↓ .<br />

Es erscheint nicht unvernünftig, dass <strong>de</strong>r Preis<br />

von <strong>de</strong>n Wahrscheinlichkeiten abhängt, die<br />

subjektiv gewählt sein können und wi<strong>de</strong>rspiegeln,<br />

ob man von einem im Mittel steigen<strong>de</strong>n<br />

o<strong>de</strong>r sinken<strong>de</strong>n Aktienkurs ausgeht, ob also<br />

a ↑ w ↑ + a ↓ w ↓ > a o<strong>de</strong>r a ↑ w ↑ + a ↓ w ↓ < a<br />

gilt (Bulle o<strong>de</strong>r Bär). Von geringem Interesse<br />

scheint dagegen das Geschehen am<br />

Aktienmarkt aus Sicht <strong>de</strong>sjenigen zu sein, <strong>de</strong>r<br />

a ↑ w ↑ + a ↓ w ↓ = a annimmt, <strong>de</strong>r es also als ein<br />

faires Glücksspiel ansieht. Wir wer<strong>de</strong>n sehen.<br />

Was ist <strong>de</strong>r korrekte Preis eines Termingeschäfts<br />

Diese Überlegungen führen nicht zum Ziel,<br />

wenn man nach <strong>de</strong>m korrekten Preis <strong>de</strong>s<br />

Kontrakts fragt. Den gibt es nämlich, unabhängig<br />

von irgendwelchen Annahmen über<br />

Wahrscheinlichkeiten. Um ihn zu fin<strong>de</strong>n,<br />

betrachten wir ein Portfolio (x,y), bestehend<br />

aus x Aktien und y auf einem Konto gelagerten<br />

Gel<strong>de</strong>inheiten (Euro). Sein zukünftiger<br />

Wert ist dann entwe<strong>de</strong>r xa ↑ + y o<strong>de</strong>r<br />

xa ↓ + y, je nach<strong>de</strong>m, wie sich <strong>de</strong>r Aktienkurs<br />

entwickelt; (<strong>de</strong>r Einfachheit halber sehen wir<br />

von einer Verzinsung <strong>de</strong>s Gel<strong>de</strong>s ab). Wählen<br />

wir insbeson<strong>de</strong>re x und y solchermaßen, dass<br />

das Gleichungssystem<br />

xa ↑ + y = t ↑ und xa ↓ + y = t ↓<br />

erfüllt ist, so ist das Portfolio in Zukunft<br />

gera<strong>de</strong> so viel wert wie unser Terminkontrakt,<br />

man spricht von Duplikation. Offenbar<br />

darf dann auch aktuell keine Wertdifferenz<br />

bestehen, <strong>de</strong>r heutige korrekte Preis <strong>de</strong>s Terminkontrakts<br />

ist also<br />

P korrekt<br />

= xa + y<br />

Diese Überlegung hat erst einmal, wie dies<br />

Black, Scholes und Merton als erste erkannten,<br />

gar nichts mit Zufall zu tun.<br />

Und <strong>de</strong>nnoch: Ein tiefer gehen<strong>de</strong>s Verständnis<br />

erwächst nur, wenn man <strong>de</strong>n Zufall doch<br />

wie<strong>de</strong>r ins Spiel bringt. Setzt man nämlich<br />

obige Wahrscheinlichkeiten speziell als<br />

w ↑ = (a–a ↓ ) / (a ↑ –a ↓ ) und<br />

w ↓ = 1 – w ↑ = (a ↑ –a) / (a ↑ –a ↓ ),<br />

so folgt, wie man durch Auflösen <strong>de</strong>s Gleichungssystems<br />

nach x und y und Einsetzen<br />

in die Formel für P korrekt<br />

ohne Weiteres nachrechnen<br />

kann,<br />

P korrekt<br />

= P(w ↑ ,w ↓ ) sowie a = a ↑ w ↑ +a ↓ w ↓ .<br />

Der Aktienmarkt als faires Glücksspiel<br />

Dieses ist nun eine Überraschung: Der korrekte<br />

Preis <strong>de</strong>s Finanzkontrakts ist <strong>de</strong>rselbe,<br />

<strong>de</strong>r sich ergibt, wenn man <strong>de</strong>n Aktienmarkt<br />

als faires Glücksspiel ansieht! Folgerichtig<br />

wird in <strong>de</strong>r Finanz<strong>mathe</strong>matik mit diesen<br />

hypothetischen Wahrscheinlichkeiten gerechnet,<br />

mögen sie nun die Wirklichkeit angemessen<br />

beschreiben o<strong>de</strong>r nicht.<br />

Diese Erkenntnisse haben die Finanz<strong>mathe</strong>matik<br />

umgekrempelt und sie heute zu<br />

einem <strong>de</strong>r wichtigsten und anspruchsvollsten<br />

Gebiete <strong>de</strong>r Stochastik wer<strong>de</strong>n lassen. Es<br />

ist nicht nur so, dass das doch recht dürre<br />

Gleichungssystem mit Wahrscheinlichkei-


N 084<br />

N 091<br />

N 071A<br />

N 072A<br />

N 071 N 072<br />

N 083<br />

N 080<br />

N 013<br />

N 070A<br />

N 073<br />

N 085<br />

N 070<br />

N 092<br />

N 081<br />

N 090<br />

N 087A<br />

E 082A<br />

E 011A<br />

N 087<br />

N 088<br />

E 085<br />

E 084<br />

N 082<br />

N 082A<br />

N 086A<br />

N 086<br />

E 082 E 081<br />

F 080<br />

E 091<br />

E 080B<br />

E 010<br />

M 090<br />

E 080C<br />

M 080<br />

M 090A<br />

M 070C<br />

M 070 M 060<br />

M 070B<br />

E 080A<br />

E 080<br />

E 002<br />

A 097<br />

A 005<br />

A 083A<br />

A 003<br />

A 080B<br />

A 080A<br />

A 085A<br />

A 009A<br />

A 009<br />

A 086<br />

A 089<br />

A 087<br />

A 083 A 082<br />

A 001<br />

A 081<br />

A 091 A 090<br />

A 080<br />

A 002<br />

A 084A<br />

A 070<br />

D 085B<br />

A 070B<br />

D 095<br />

B 082A<br />

D 085A<br />

B 082<br />

-<br />

-<br />

B 002<br />

D 085<br />

B 081<br />

D 081<br />

D 080<br />

D 073A D 073<br />

D 072A D 072<br />

D 082<br />

D 019A<br />

B 071<br />

B 070<br />

B 070A<br />

B 071A<br />

D 090<br />

B 080<br />

C 082<br />

D 083<br />

31<br />

ten eine plastische Interpretation erfährt. Die<br />

Einsicht, dass Termingeschäfte korrekterweise<br />

(sicher nicht als faire Spiele betrachtet, aber)<br />

so analysiert wer<strong>de</strong>n können, als wären sie<br />

faire Spiele, ermöglicht <strong>de</strong>n Einsatz ausgefeilter<br />

Techniken aus <strong>de</strong>r Stochastik, wie sie die<br />

Martingaltheorie bereitstellt.<br />

Finanz<strong>mathe</strong>matiker wer<strong>de</strong>n heute in <strong>de</strong>n<br />

Finanzinstituten allerorten gebraucht, in einer<br />

Zahl, wie sie die Universitäten bisher kaum<br />

bereitstellen können. Darauf muss man sich<br />

einstellen – auch wenn man <strong>de</strong>n Finanzmärkten<br />

mit all ihren Turbulenzen reserviert<br />

gegenübersteht.<br />

Diese Beispiele belegen, dass <strong>de</strong>r Zufall besser<br />

ist als sein Ruf. Stochastische Mo<strong>de</strong>lle schaffen<br />

in vielen Fällen eine soli<strong>de</strong> Grundlage,<br />

die uns hilft, komplexe Zusammenhänge zu<br />

beurteilen o<strong>de</strong>r Risiken schwer vorhersehbarer<br />

Entwicklungen einzuschätzen.<br />

Literatur<br />

Götz Kersting und Anton Wakolbinger: Elementare<br />

Stochastik, Birkhäuser 2008.<br />

Odo Marquard: Apologie <strong>de</strong>s Zufälligen, Reclam 1986.<br />

David Ruelle: Zufall und Chaos, Springer 1992.<br />

Die Redaktion von „<strong>mathe</strong>-<strong>lmu</strong>.<strong>de</strong>“ bedankt sich bei Prof.<br />

Dr. Götz Kersting und bei „Forschung Frankfurt“ für die<br />

Erlaubnis, diesen Artikel erneut zu veröffentlichen.<br />

Lageplan für Mathematik am Samstag<br />

Eingang Amalienstraße<br />

M 018<br />

M 014<br />

M 010<br />

B 011<br />

M 022<br />

N 017<br />

N 019 N 021<br />

M 022A<br />

M 011<br />

M 009<br />

M 007<br />

M 005<br />

M 003<br />

M 001<br />

A 087A A 087B A 096<br />

N 015<br />

N 015A<br />

N 020<br />

M 015<br />

B 004 B 006 B 008<br />

A 027 A 088<br />

N 015B<br />

N 011<br />

N 018<br />

N 012<br />

Hörsaal M 010<br />

Hörsaal A 021<br />

A 021<br />

A 071B A 071<br />

A 030<br />

A 071A A 070A<br />

A 028<br />

C 023<br />

C 021<br />

N 009<br />

N 010<br />

A 022<br />

A 017<br />

A 020<br />

C 019<br />

N 007<br />

N 008<br />

C 017<br />

C 015<br />

N 003<br />

N 006<br />

A 015<br />

A 093A A 093<br />

A 093B<br />

A 016<br />

C 013<br />

N 004<br />

N 002<br />

C 009<br />

A 011<br />

A 085 A 084<br />

N 030<br />

A 085B A 084B<br />

A 014<br />

C 007<br />

N 032A<br />

N 032<br />

N 033<br />

C 005<br />

E 019 E 086<br />

E 020 E 018<br />

A 007<br />

A 004<br />

E 017<br />

E 016<br />

E 014<br />

E 015<br />

E 004<br />

D 001 D 003<br />

D 007<br />

D 011 D 013<br />

D 005 D 009<br />

D 017 D 019 D 080A<br />

E 013 E 083 E 012<br />

E 006<br />

E 011<br />

F 002<br />

F 008<br />

Haupteingang Geschwister-Scholl-Platz<br />

D 020


32<br />

Wie könnten Sie Ihrer Karriere Flügel verleihen<br />

Wenn Sie sich <strong>de</strong>n großen Herausfor<strong>de</strong>rungen <strong>de</strong>r Welt stellen<br />

In<strong>de</strong>m Sie dabei helfen, Satelliten ins All zu schicken<br />

In Brainstormings mit Ingenieuren, Mathematikern und Risikomanagern<br />

Mit neuen I<strong>de</strong>en zur Eindämmung <strong>de</strong>s Ozonlochs<br />

Durch alle <strong>de</strong>r genannten Punkte<br />

Unsere Stärke liegt im Wissen unserer interdisziplinären Teams. Wissen,<br />

mit <strong>de</strong>m wir komplexe Herausfor<strong>de</strong>rungen aus allen Bereichen <strong>de</strong>r Wirtschaft<br />

und Gesellschaft meistern, von Großbauprojekten über <strong>de</strong>n Klimawan<strong>de</strong>l<br />

bis hin zur Raumfahrt. So entwickeln wir maßgeschnei<strong>de</strong>rte Lösungen für<br />

Risiken, die die Menschheit heute und in Zukunft beschäftigen. Wenn Sie<br />

gemeinsam mit 10.500 Kollegen Projekte von globaler Tragweite bewegen<br />

wollen, freuen wir uns über Ihre Bewerbung.<br />

Was wir im Rahmen unseres Traineeprogramms von Ihnen erwarten und<br />

welche Chancen wir Ihnen bieten, erfahren Sie unter<br />

munichre.com/karriere

Hurra! Ihre Datei wurde hochgeladen und ist bereit für die Veröffentlichung.

Erfolgreich gespeichert!

Leider ist etwas schief gelaufen!